You are on page 1of 168

TRƯỜNG CAO ĐẲNG CÔNG NGHỆ THỦ ĐỨC

TRUNG TÂM ĐÀO TẠO NGUỒN NHÂN LỰC

TÀI LIỆU ÔN THI CHUẨN ĐẦU RA


VOL 7

Tp. Hồ Chí Minh, tháng 05 năm 2021


LƯU HÀNH NỘI BỘ

1
TOEIC TEST FORMAT
Part 1: Picture description 10 questions
Part 2: Question - Response 30 questions
Listening test
Part 3: Conversations 30 questions
Part 4: Short talks 30 questions
45 minutes TOTAL 100 questions
Part 5: Incomplete sentences 40 questions
Reading test Part 6: Text completion 12 questions
Part 7: Reading comprehension 48 questions
75 minutes TOTAL 100 questions

2
LISTENING TEST
_________________________________

In the first section of the TOEIC test, you will be tested on how well you understand
spoken English. There are four parts to this section with special directions for each
part:
Part 1 Photos
Part 2 Question – Response
Part 3 Conversations
Part 4 Talks
In this part of the TOEIC test, you will learn strategies to help you on the Listening
Comprehension section. Each part contains strategies and practice questions.
NOTE: The TOEIC test directions for each part of the TOEIC test will be given at the
beginning of the section. Read the directions carefully to be sure you understand them.

3
PART 1: PICTURE DESCRIPTION
LISTENING TEST
In the Listening test, you will be asked to demonstrate how well you understand spoken
English. The entire Listening test will last approximately 45 minutes. There are four parts,
and directions are given for each part. You must mark your answers on the separate answer
sheet.

PART 1
Directions: For each question in this part, you will hear four statements about a picture in
your test book. When you hear the statement, you must select the one statement that best
describes what you see in the picture. Then find the number of the question on your answer
sheet and mark your answer. The statements will not be printed in your test book and will be
spoken only one time.

Example

Statement (B), “He’s making a presentation.” is the best description of the picture, so you
should select answer (B) and mark it on your answer sheet.

4
STRATEGY OVERVIEW

LANGUAGE STRATEGIES
There are two types of photographs you will see on Part 1 of the TOEIC test:
 photos of people
 photos of things
In Part 1 if you see photos of people, you will hear statements that may answer these
questions about the people:
 Who are they?
 Where are they?
 What are they doing?
 What do they look like?
In part 1, if you see photos of things, you will hear statements that may answer these
questions about the things:
 What are they?
 Where are they?
 What was done to them?
 What do they look like?

TEST STRATEGIES
Some answer choices are designed to trick you. They are written to seem like the correct
answer. You must learn to recognize the way the answer choices may seem correct:
 Some choices have words that sound similar to the correct answer.
 Some choices have words related to the correct answer.
 Some choices have words used in a different context.
 Some choices have incorrect details.
 Some choices make incorrect inferences.
The two examples on the following pages will help you develop test strategies to avoid
answer choices that seem correct.

5
(A) The carpenter is hammering a nail. (correct answer)
(B) The snail is crawling up the wall. (similar sound snail/nail)
(C) The carpet is nailed to the wall. (similar sound carpet/carpenter; nail and wall
used in a different context)
(D) The handyman is putting away his tools. (handyman and tools related to correct answer)

(A) The tourist is buying some postcards. (tourist used in a different context)
(B) The passenger is checking his bags. (incorrect inference)
(C) The traveler is pushing his luggage (incorrect detail)
ahead of him.
(D) The man is pulling his suitcase behind (correct answer)
him.

6
PHOTOS OF PEOPLE

LANGUAGE STRATEGIES
You will hear statements that may answer these questions:
 Who are they?
 Where are they?
 What are they doing?
 What do they look like?

TEST STRATEGIES
You will hear statements that may seem correct.
 Some choices have words that sound similar to the correct answer.
 Some choices have words related to the correct answer.
 Some choices have words used in a different context.
 Some choices have incorrect details.
 Some choices make incorrect inferences.
Use these strategies when you do the exercises. They will help you choose the
right answer.

PHOTOS OF THINGS

LANGUAGE STRATEGIES
You will hear statements that may answer these questions:
 What are they?
 Where are they?
 What was done to them?
 What do they look like?

TEST STRATEGIES
You will hear statements that may seem correct.
 Some choices have words that sound similar to the correct answer.
 Some choices have words related to the correct answer.
 Some choices have words used in a different context.
 Some choices have incorrect details.
 Some choices make incorrect inferences.
Use these strategies when you do the exercises. They will help you choose the
right answer.

7
PRACTICE

Directions: For each question in this part, you will hear four statements about a picture in your test
book. When you hear the statement, you must select the one statement that best describes what you
see in the picture. Then find the number of the question on your answer sheet and mark your
answer. The statements will not be printed in your test book and will be spoken only one time.

PRACTICE 1

8
9
10
11
12
PRACTICE 2

13
14
15
16
17
PRACTICE 3

18
19
20
21
22
23
PRACTICE 4

24
25
26
27
28
PRACTICE 5

29
30
31
32
33
PRACTICE 6

34
35
36
37
38
PRACTICE 7

39
40
41
42
43
PRACTICE 8

44
45
46
47
48
PRACTICE 9

49
50
51
52
53
PRACTICE 10

54
55
56
57
58
PART 2: QUESTION - RESPONSE
PART 2
Directions: You will hear a question or statement and three responses spoken in English.
They will not be printed in your test book and will be spoken only one time. Select the best
response to the question or statement and mark the letter (A), (B), or (C) on your answer
sheet.

Example

You will hear: Where is the meeting room?


You will also hear: (A) To meet the new director.
(B) It’s the first room on the right.
(C) Yes, it’s 2 o’clock.

The best response to the question “Where is the meeting room?” is choice (B). “It’s the first
room on the right.” So (B) is the correct answer. You should mark answer (B) on your
answer sheet.

STRATEGY OVERVIEW

LANGUAGE STRATEGIES
In this part, you will learn how certain words will help you identify the purpose of a
question. These are the purposes you will learn about:
 identifying time
 identifying people
 identifying an opinion
 identifying a choice
 identifying a suggestion
 identifying a reason
 identifying a location

59
LANGUAGE STRATEGIES
In Part 2, like in Part 1, some answer choices are designed to trick you. You must learn
to recognize the way the answer choices may seem correct:
 Some choices have words that sound similar to the correct answer.
 Some choices have words related to the correct answer.
 Some choices have words used in a different context.
 Some choices use an incorrect verb tense or person.
 Some choices are an inappropriate response to the type of question.
Look at these examples:

Example 1
Mrs. Smith is never late for her meeting.
(A) He is always on time. (incorrect person)
(B) Her meeting is running late. (words used in a different context)
(C) She’s very punctual. (correct answer)

Example 2
Are you hiring an assistant or a secretary?
(A) I need both. (correct answer)
(B) Yes, I am. (inappropriate response to an “or” question)
(C) I was hired yesterday. (incorrect verb tense)

Example 3
How can I get to the train station from here?
(A) The radio station is a block away. (words used in a different context)
(B) You can take the A-4 bus. (correct answer)
(C) The plane leaves in ten minutes. (words that sound similar)

60
PRACTICE 1
PART 2
Directions: You will hear a question or statement and three responses spoken in English.
They will not be printed in your test book and will be spoken only one time. Select the best
response to the question or statement and mark the letter (A), (B), or (C) on your answer
sheet.

Example
You will hear: Where is the meeting room?
You will also hear: (A) To meet the new director.
(B) It’s the first room on the right.
(C) Yes, it’s 2 o’clock.

The best response to the question “Where is the meeting room?” is choice (B). “It’s the first
room on the right.” So (B) is the correct answer. You should mark answer (B) on your answer
sheet.
11. (A) (B) (C) 26. (A) (B) (C)
12. (A) (B) (C) 27. (A) (B) (C)
13. (A) (B) (C) 28. (A) (B) (C)
14. (A) (B) (C) 29. (A) (B) (C)
15. (A) (B) (C) 30. (A) (B) (C)
16. (A) (B) (C) 31. (A) (B) (C)
17. (A) (B) (C) 32. (A) (B) (C)
18. (A) (B) (C) 33. (A) (B) (C)
19. (A) (B) (C) 34. (A) (B) (C)
20. (A) (B) (C) 35. (A) (B) (C)
21. (A) (B) (C) 36. (A) (B) (C)
22. (A) (B) (C) 37. (A) (B) (C)
23. (A) (B) (C) 38. (A) (B) (C)
24. (A) (B) (C) 39. (A) (B) (C)
25. (A) (B) (C) 40. (A) (B) (C)

61
PRACTICE 2
PART 2
Directions: You will hear a question or statement and three responses spoken in English.
They will not be printed in your test book and will be spoken only one time. Select the best
response to the question or statement and mark the letter (A), (B), or (C) on your answer
sheet.

Example
You will hear: Where is the meeting room?
You will also hear: (A) To meet the new director.
(B) It’s the first room on the right.
(C) Yes, it’s 2 o’clock.

The best response to the question “Where is the meeting room?” is choice (B). “It’s the first
room on the right.” So (B) is the correct answer. You should mark answer (B) on your answer
sheet.
11. (A) (B) (C) 26. (A) (B) (C)
12. (A) (B) (C) 27. (A) (B) (C)
13. (A) (B) (C) 28. (A) (B) (C)
14. (A) (B) (C) 29. (A) (B) (C)
15. (A) (B) (C) 30. (A) (B) (C)
16. (A) (B) (C) 31. (A) (B) (C)
17. (A) (B) (C) 32. (A) (B) (C)
18. (A) (B) (C) 33. (A) (B) (C)
19. (A) (B) (C) 34. (A) (B) (C)
20. (A) (B) (C) 35. (A) (B) (C)
21. (A) (B) (C) 36. (A) (B) (C)
22. (A) (B) (C) 37. (A) (B) (C)
23. (A) (B) (C) 38. (A) (B) (C)
24. (A) (B) (C) 39. (A) (B) (C)
25. (A) (B) (C) 40. (A) (B) (C)

62
PRACTICE 3
PART 2
Directions: You will hear a question or statement and three responses spoken in English.
They will not be printed in your test book and will be spoken only one time. Select the best
response to the question or statement and mark the letter (A), (B), or (C) on your answer
sheet.

Example
You will hear: Where is the meeting room?
You will also hear: (A) To meet the new director.
(B) It’s the first room on the right.
(C) Yes, it’s 2 o’clock.

The best response to the question “Where is the meeting room?” is choice (B). “It’s the first
room on the right.” So (B) is the correct answer. You should mark answer (B) on your answer
sheet.
11. (A) (B) (C) 26. (A) (B) (C)
12. (A) (B) (C) 27. (A) (B) (C)
13. (A) (B) (C) 28. (A) (B) (C)
14. (A) (B) (C) 29. (A) (B) (C)
15. (A) (B) (C) 30. (A) (B) (C)
16. (A) (B) (C) 31. (A) (B) (C)
17. (A) (B) (C) 32. (A) (B) (C)
18. (A) (B) (C) 33. (A) (B) (C)
19. (A) (B) (C) 34. (A) (B) (C)
20. (A) (B) (C) 35. (A) (B) (C)
21. (A) (B) (C) 36. (A) (B) (C)
22. (A) (B) (C) 37. (A) (B) (C)
23. (A) (B) (C) 38. (A) (B) (C)
24. (A) (B) (C) 39. (A) (B) (C)
25. (A) (B) (C) 40. (A) (B) (C)

63
PRACTICE 4
PART 2
Directions: You will hear a question or statement and three responses spoken in English.
They will not be printed in your test book and will be spoken only one time. Select the best
response to the question or statement and mark the letter (A), (B), or (C) on your answer
sheet.

Example
You will hear: Where is the meeting room?
You will also hear: (A) To meet the new director.
(B) It’s the first room on the right.
(C) Yes, it’s 2 o’clock.

The best response to the question “Where is the meeting room?” is choice (B). “It’s the first
room on the right.” So (B) is the correct answer. You should mark answer (B) on your answer
sheet.
11. (A) (B) (C) 26. (A) (B) (C)
12. (A) (B) (C) 27. (A) (B) (C)
13. (A) (B) (C) 28. (A) (B) (C)
14. (A) (B) (C) 29. (A) (B) (C)
15. (A) (B) (C) 30. (A) (B) (C)
16. (A) (B) (C) 31. (A) (B) (C)
17. (A) (B) (C) 32. (A) (B) (C)
18. (A) (B) (C) 33. (A) (B) (C)
19. (A) (B) (C) 34. (A) (B) (C)
20. (A) (B) (C) 35. (A) (B) (C)
21. (A) (B) (C) 36. (A) (B) (C)
22. (A) (B) (C) 37. (A) (B) (C)
23. (A) (B) (C) 38. (A) (B) (C)
24. (A) (B) (C) 39. (A) (B) (C)
25. (A) (B) (C) 40. (A) (B) (C)

64
LANGUAGE STRATEGIES
In the exercises for Part 2, you learned how certain words would help you identify the
purpose of a question. Knowing these words will help you choose the right answer.
Listen for words that identify:
 time, such as when, how long, what time, etc.
 people, such as who, whose, who's, etc.
 an opinion, such as what, how, why, etc.
 a choice, such as what, which, or, etc.
 a suggestion, such as why don’t we, let's, what about, etc.
 a reason, such as why don't we, excuse, reason, etc.
 a location, such as what, where, how far, etc.

65
PART 3: CONVERSATIONS
PART 3
Directions: You will hear some conversations between two people. You will be asked to
answer three questions about what the speakers say in each conversation. Select the best
response to each question and mark the letter (A), (B), (C), or (D) on your answer sheet.
The conversations will not be printed in your test book and will be spoken only one time.

STRATEGY OVERVIEW

LANGUAGE STRATEGIES
In this part, you will learn how certain words will help you identify the purpose of a
question. These are the purposes you will learn about:
 identifying time
 identifying people
 identifying an intent
 identifying the topic
 identifying a reason
 identifying a location
 identifying an opinion
 identifying stress and tone
The words you listen in this part will help you develop strategies to choose the correct
answer.

TEST STRATEGIES
In Part 3, like in Parts 1 and 2, some answer choices are designed to trick you. You must
learn to recognize the way the answer choices may seem correct.
 Some choices have words that sound similar to the correct answer.
 Some choices have words like the correct answer, but with a different meaning.
 Some choices have words used in a different context.
 Some choices have incorrect details.
 Some choices have incorrect inferences.
 Some choices have irrelevant details.
Look at these examples:

66
Example 1
SPEAKER A: The prices at this restaurant are low, but the food's always cold.
SPEAKER B: I know, and the people who work here aren’t very friendly.
SPEAKER A: Let’s eat somewhere else.
What do they say about the restaurant?
(A) The people are friendly. (incorrect detail)
(B) The prices are low. (correct answer)
(C) The food is old. (sounds like correct answer)
(D) The service is slow. (sounds like correct answer)

Example 2
SPEAKER A: I’d like to book a flight to Santiago.
SPEAKER B: Certainly. When would you like to fly?
SPEAKER A: Next Monday. Can you make a hotel reservation for me, also?
What is Speaker B’s occupation?
(A) Travel agent (correct answer)
(B) Pilot (incorrect inference)
(C) Hotel manager (incorrect detail)
(D) Librarian (incorrect inference)

Example 3
SPEAKER A: How’s your accounting class, Marvin?
SPEAKER B: The teacher’s interesting, but the work is really hard.
SPEAKER A: Well, at least you’re not bored.
What does Marvin say about his class?
(A) It’s boring. (incorrect detail)
(B) The chairs are hard. (word used in a different context)
(C) There’s too much work. (incorrect detail)
(D) The teacher is interesting. (correct answer)

67
PRACTICE 1

PART 3
Directions: You will hear some conversations between two people. You will be asked to
answer three questions about what the speakers say in each conversation. Select the best
response to each question and mark the letter (A), (B), (C), or (D) on your answer sheet.
The conversations will not be printed in your test book and will be spoken only one time.

41. Where most likely does the conversation take place?


(A) At a grocery store
(B) At a hotel
(C) At a travel agency
(D) At an airport
42. What does the man say he can do for the woman?
(A) Store her luggage
(B) Make a reservation
(C) Arrange transportation
(D) Print out a receipt
43. What does the man give the woman?
(A) A parking pass
(B) A list of restaurants
(C) A city map
(D) A travel brochure
_______________________________
44. What are the speakers mainly discussing?
(A) A job transfer
(B) A trip itinerary
(C) A store opening
(D) An advertising campaign
45. What most likely is the woman's job?
(A) Human resources manager
(B) Real estate agent
(C) Sales team leader
(D) Computer programmer
46. What will the man give the woman?
(A) A business card
(B) An e-mail address
(C) A rental agreement
(D) A resume
_______________________________
47. What will take place on Friday?
(A) An office will be painted.
(B) A carpet will be replaced.
(C) Computers will be upgraded.
(D) Work assignments will be distributed.

68
48. What is the woman concerned about?
(A) The availability of a staff member
(B) The size of a meeting space
(C) The accessibility of some files
(D) The deadline for a project
49. Who will the man contact?
(A) The training coordinator
(B) The security office
(C) The maintenance supervisor
(D) The technical support department
______________________________
50. Why is the woman calling?
(A) To discuss a seating plan
(B) To arrange catering services
(C) To invite the man to speak at a banquet
(D) To inquire about flower arrangements
51. What will the woman provide?
(A) An event schedule
(B) Delivery instructions
(C) Menu options
(D) A photograph
52. How much advance notice does the man require?
(A) Two days
(B) One week
(C) Two weeks
(D) One month
_______________________________
53. According to the woman, what will happen this summer?
(A) A cooking course will be offered.
(B) A line of cookware will be launched.
(C) A cafe will open.
(D) A documentary will be filmed.
54. What is Steven Okada known for?
(A) Owning a chain of stores
(B) Writing a magazine column
(C) Founding a cooking school
(D) Hosting a television show
55. What does the woman recommend?
(A) Requesting an interview
(B) Registering soon
(C) Subscribing to a newsletter
(D) Asking about cancellations
_______________________________
56. Where does the conversation most likely take place?
(A) At a factory
(B) At a home improvement store
(C) At a construction site
69
(D) At a bus station
57. What does the woman want to know?
(A) How much a project will cost
(B) What part is needed
(C) Why a problem occurred
(D) When a repair will be completed
58. What will the man probably do next?
(A) Inform his supervisor
(B) Review an order form
(C) Check the inventory
(D) Schedule a technician
______________________________
59. What are the speakers mainly discussing?
(A) A budget surplus
(B) A news report
(C) A conference agenda
(D) A software upgrade
60. What has Ms. Han asked employees to do?
(A) Sign up for training
(B) Update their calendars
(C) Review some invoices
(D) Recommend projects
61. What does the man suggest?
(A) Purchasing equipment
(B) Consulting colleagues
(C) Holding monthly meetings
(D) Reducing spending
______________________________
62. What is the purpose of the telephone call?
(A) To describe a service
(B) To publicize an event
(C) To conduct a survey
(D) To request a proposal
63. What does the woman say she has done?
(A) Used public transportation
(B) Organized a neighborhood group
(C) Followed news reports
(D) Contacted city officials
64. What does the man suggest the woman do?
(A) Write a newspaper article
(B) Complete a form online
(C) Speak with an expert
(D) Order a free sample
______________________________
65. Who most likely is the man?
(A) An actor
(B) A set designer
70
(C) A producer
(D) A tailor
66. What problem does the man mention?
(A) He cannot find some papers.
(B) He has a conflicting work obligation.
(C) Some promotional materials are not ready.
(D) An audition has been postponed.
67. What does the man offer to do next Wednesday?
(A) Introduce a speaker
(B) Pick up a script
(C) Take some pictures
(D) Meet with a designer
_______________________________
68. What is the man calling to tell the woman?
(A) Her article has been accepted for publication.
(B) She has been selected to receive a grant.
(C) Research assistants have been hired.
(D) A deadline has been extended.
69. What has the woman recently done?
(A) Expanded her laboratory
(B) Interviewed for a job
(C) Presented her research
(D) Organized a conference
70. What does the man ask the woman to do?
(A) Check some data
(B) Sign a contract
(C) Submit letters of reference
(D) Revise a submission
______________________________
PRACTICE 2

PART 3
Directions: You will hear some conversations between two people. You will be asked to
answer three questions about what the speakers say in each conversation. Select the best
response to each question and mark the letter (A), (B), (C), or (D) on your answer sheet.
The conversations will not be printed in your test book and will be spoken only one time.

41. Where are the speakers?


(A) At a movie theater
(B) At a furniture store
(C) At a restaurant
(D) At an outdoor concert
42. Why is the woman in a hurry?
(A) She has to go back to work.
(B) She has to catch a bus.

71
(C) She is meeting a friend shortly.
(D) She is going to see a show.
43. What does the man say he will do?
(A) Give the woman a discount
(B) Write up an order
(C) Speak with another employee
(D) Exchange a ticket
______________________________
44. What did the man leave in the study room?
(A) A computer
(B) A set of keys
(C) A book
(D) A wallet
45. What information does the woman ask for?
(A) The man's name
(B) The value of an item
(C) The number of a room
(D) The title of a book
46. What will the woman probably do next?
(A) Contact security
(B) Unlock a door
(C) File a report
(D) Check a database
______________________________
47. Why does the man want to change his ticket?
(A) He needs time to make a phone call-
IB) His destination has changed.
(C) He arrived at the station early.
(D) He wants to upgrade his seat.
48. What does the woman say about the four o'clock train?
(A) It has been delayed.
(B) It costs more.
(C) It is fully booked.
(D) It will not stop in Baltimore.
49. When will the man probably catch his train
(A) At 3:00 P.M.
(B) At 3:30 P.M.
(C) At 4:30 P.M.
(D) At 5:00 P.M.
______________________________
50. Why did the woman miss the staff meeting?
(A) She was stuck in traffic.
(B) She was with a client.
(C) She was finishing a project.
(D) She had a doctor's appointment.
51. What is the man scheduling?
(A) Telephone-marketing workshops
72
(B) New-employee orientations
(C) Department budget discussions
(D) Software-training sessions
52. What will the man send to attendees?
(A) A product brochure
(B) A meeting agenda
(C) An employee handbook
(D) A travel itinerary
______________________________
53. Where does the woman work?
(A) At an art-supply store
(B) At a university
(C) At a photography studio
(D) At a community center
54. What is important to the man about the class?
(A) It should take place in the evening.
(B) It should be taught at a beginner level.
(C) It should be taught by a particular instructor.
(D) It should have a reasonable price.
55. What does the woman suggest the man do?
(A) Purchase materials
(B) Meet with an instructor
(C) Submit a portfolio
(D) Fill out a registration form
______________________________
56. What is the man working on?
(A) A board presentation
(B) A building plan
(C) A purchase order
(D) A financial report
57. Why is it noisy in the building?
(A) Some furniture is being moved.
(B) A cleaning crew is working.
(C) A space is under construction.
(D) A large group of clients is visiting.
58. What does the woman offer to do?
(A) Speak with the maintenance staff
(B) Help the man revise a handout
(C) Postpone a meeting
(D) Lend the man some equipment
______________________________
59. What does the woman ask about the farm?
(A) How much a ticket costs
(B) Whether it accepts large groups
(C) When its guided tours begin
(D) Where it is located
60. What does the man offer the woman?
73
(A) A choice of activities
(B) An alternate date
(C) A group discount
(D) A preferred parking location
61. What does the woman say she must do before she calls back?
(A) Consult the tour participants
(B) Get approval for expenses
(C) Arrange transportation
(D) Check some references
______________________________
62. Where does the woman work?
(A) At an employment agency
(B) At an insurance company
(C) At an Internet service provider
(D) At a department store
63. Why does the man want to call the woman back?
(A) He is late for a seminar.
(B) He is at a client's office.
(C) He is expecting another phone call.
(D) He is on his way to the airport.
64. What does the woman want the man to review?
(A) An annual report
(B) An applicant's resume
(C) A job description
(D) An employment contract
______________________________
65. Where most likely do the speakers work?
(A) At a mail-delivery service
(B) At a moving company
(C) At a catering business
(D) At a car rental agency
66. What is the problem?
(A) No vehicles are available.
(B) A delivery was canceled.
(C) Some supplies are missing.
(D) An employee is late for work.
67. What will the man probably do next?
(A) Visit a customer
(B) Call a coworker
(C) Unload a shipment
(D) Repair a van
______________________________
68. What type of business is the woman calling from?
(A) A construction company
(B) A sporting-goods manufacturer
(C) An advertising firm
(D) A printing shop
74
69. What problem is the woman calling about?
(A) An invoice showed the wrong amount.
(B) A printer did not produce enough copies.
(C) A magazine was sent to the wrong address.
(D) An article contained an error.
70. What does the man say he will do?
(A) Approve a refund
(B) Arrange an interview
(C) Find a photograph
(D) Expedite a delivery

PRACTICE 3

PART 3
Directions: You will hear some conversations between two people. You will be asked to
answer three questions about what the speakers say in each conversation. Select the best
response to each question and mark the letter (A), (B), (C), or (D) on your answer sheet.
The conversations will not be printed in your test book and will be spoken only one time.

41. What is the woman planning to do after


work?
(A) Buy a new appliance
(B) Attend a business dinner
(C) Pick up a car
(D) Visit a friend
42. What information does the man provide?
(A) Where to buy a ticket
(B) Where to catch a bus
(C) Where to find a taxi
(D) Where to locate an office
43. What problem does the woman mention?
(A) She does not have enough money.
(B) Her car is being repaired.
(C) She is late for a meeting.
(D) There is too much traffic.
______________________________
44. What is the woman looking for?
(A) Envelopes
(B) Invitations
(C) Mailing labels
(D) Printer paper
45. What is the woman organizing?
(A) A birthday party
(B) A company picnic
(C) An awards dinner
75
(D) A retirement celebration
46. What does the man offer to do?
(A) Recommend some equipment
(B) Look up some information
(C) Show the woman some samples
(D) Mail some packages
______________________________
47. Why is the man calling?
(A) To ask for directions
(B) To reserve a room
(C) To confirm an order
(D) To cancel an appointment
48. Where does the woman most likely work?
(A) At a restaurant
(B) At a shipping company
(C) At a convention center
(D) At a travel agency
49. What will the man leave at the front desk on Monday?
(A) An invoice for printing services
(B) A payment for some food
(C) An agenda for a meeting
(D) A package of books
______________________________
50. What has the woman lost?
(A) A discount coupon
(B) A product code
(C) A credit card
(D) A gift receipt
51. According to the man, what can the woman receive?
(A) Reduced prices
(B) Free gift wrapping
(C) A store credit
(D) A product sample
52. What will the woman probably do next?
(A) Check a catalog
(B) Look at some merchandise
(C) Fill out a form
(D) Send a package
______________________________
53. What is the man unable to do?
(A) Print out a report
(B) Shut down his computer
(C) Make an appointment
(D) Access the Internet
54. What does the man say he must do?
(A) Revise a schedule
(B) E-mail some documents
76
(C) Meet with clients
(D) Telephone a colleague
55. What does the woman suggest?
(A) Using another computer
(B) Rescheduling a meeting
(C) Upgrading some equipment
(D) Working on another project
______________________________
56. How did the woman learn about the man's agency?
(A) From a neighbor
(B) From a magazine
(C) From a coworker
(D) From the Internet
57. What does the woman say she might do before selling her house?
(A) Inspect some other properties
(B) Make some improvements
(C) Attend a real-estate seminar
(D) Place an advertisement
58. What will the man do when they meet?
(A) Recommend specific changes
(B) Provide a list of references
(C) Take photographs
(D) Sign a contract
______________________________
59. What are the speakers planning to do?
(A) Meet some clients
(B) Shop for phones
(C) Eat at a restaurant
(D) Attend a theater performance
60. Why are the speakers unable to contact Juliana?
(A) She is rehearsing for a performance.
(B) She has turned off her mobile phone.
(C) She is traveling overseas.
(D) She is teaching a seminar.
61. Why is the man relieved?
(A) Their friend already has a ticket.
(B) Ticket prices have not increased.
(C) They found a parking space nearby.
(D) Good seats are still available.
______________________________
62. What has Mr. Moreno recently done?
(A) Won a cooking competition
(B) Given a cooking demonstration
(C) Opened a restaurant
(D) Written a cookbook
63. What is special about Mr. Moreno's recipes?
(A) They are easy to prepare.
77
(B) They use healthy ingredients.
(C) They are mostly vegetarian.
(D) They are old family recipes.
64. What does Mr. Moreno say about Madrid?
(A) He was born there.
(B) He usually goes there on vacation.
(C) He has a restaurant there.
(D) He is on a television program there.
______________________________
65. Where do the speakers most likely work?
(A) At a medical-supply store
(B) At a research institute
(C) At a security firm
(D) At a community hospital
66. What will the man ask the security office to give the woman?
(A) A storage-room key
(B) A parking permit
(C) A tour of the building
(D) A list of safety procedures
67. What does the woman say she will do in the afternoon?
(A) Make copies of a report
(B) Conduct an experiment
(C) Obtain an identification card
(D) Pick up some work supplies
______________________________
68. Why is the man calling?
(A) To make a job offer
(B) To request an interview
(C) To confirm a deadline
(D) To describe an apartment
69. What does the woman ask for?
(A) A larger office
(B) A revised contract
(C) More time to make a decision
(D) Additional staff to complete a project
70. What will the man send the woman?
(A) A map of the city
(B) A list of moving companies
(C) Directions to a facility
(D) Information about benefits

78
LANGUAGE STRATEGIES
In the exercises for Part 3, you learned how certain words would help you identify the
purpose of a question. Knowing these words will help you choose the right answer.
Listen for words that identify:
 time, such as when, how long, what time, etc.
 people, such as who, whose, etc.
 intent, such as plan, going to, will, etc.
 the topic, such as talking about, discussing, about, etc.
 a reason, such as why
 a location, such as where
 an opinion, such as think of, opinion, say about, believe, etc.
Remember to listen for tone and stress to identify meaning.

79
PART 4: TALKS
PART 4
Directions: You will hear some short talks given by a single speaker. You will be asked to
answer three questions about what the speaker says in each short talk. Select the best
response to each question and mark the letter (A), (B), (C), or (D) on your answer sheet.
The talks will not be printed in your test book and will be spoken only one time.

STRATEGY OVERVIEW

LANGUAGE STRATEGIES
In this part, you will learn how certain words will help you identify the purpose of a
question. These are the purposes you will learn about:
 identifying sequence
 identifying the audience
 identifying a location
 identifying the topic
 identifying a request
The words you listen in this part will help you develop strategies to choose the correct
answer.

TEST STRATEGIES
In Part 4, like in Parts 1, 2 and 3, some answer choices are designed to trick you. You
must learn to recognize the way the answer choices may seem correct.
 Some choices have words that sound similar to the correct answer.
 Some choices use words related to the correct answer.
 Some choices have words like the correct answer, but with a different meaning.
 Some choices have words used in a different context.
 Some choices have incorrect details.
 Some choices have incorrect inferences.

80
Look at these examples:
Example 1
You will hear:
“Bildo’s Department Store has openings for cashiers, management trainees, and
buyer’s assistants. Call 555-2121 to apply or send your résumé to 152 South State
Street.”

For one of the three questions, you might hear:


Who is this advertisement for?
(A) Job seekers (correct answer)
(B) Shoppers (incorrect inference)
(C) Train passengers (word used with a different context)
(D) Employers (word related to correct answer)

Example 2
You will hear:
“Next Tuesday is Library Forgiveness Day. All overdue books and late fines are
forgiven. Return your overdue books to the library on Tuesday and you won’t be
charged a late fine.”

For one of the three questions, you might hear:


What are library users asked to do next Tuesday?
(A) Pay a fine. (incorrect detail)
(B) Return overdue books. (correct answer)
(C) Give new books to the library. (sound like correct answer)
(D) Charge their fines to their credit card. (incorrect detail)

81
PRACTICE 1

PART 4
Directions: You will hear some short talks given by a single speaker. You will be asked to
answer three questions about what the speaker says in each short talk. Select the best
response to each question and mark the letter (A), (B), (C), or (D) on your answer sheet.
The talks will not be printed in your test book and will be spoken only one time.

71. Where does the speaker most likely work?


(A) At an insurance company
(B) At a medical office
(C) At a travel agency
(D) At a fitness center
72. What does the speaker ask the listener to bring?
(A) A passport
(B) An insurance card
(C) A doctor's note
(D) A payment
73. According to the speaker, what requires 24-hour advance notice?
(A) Appointment changes
(B) Prescription orders
(C) Room reservations
(D) Flight cancellations
______________________________
74. What is the report mainly about?
(A) A community fund-raiser
(B) A sporting event
(C) A cooking demonstration
(D) A talent show
75. What will event attendees be able to do?
(A) Purchase a video of the event
(B) Meet local business leaders
(C) Vote for a performer
(D) Receive free merchandise
76. What are event attendees advised to do?
(A) Arrive at the location early
(B) Buy tickets in advance
(C) Bring their own chairs
(D) Check a schedule online
______________________________
77. What item did the speaker order?
(A) A backpack
(B) A ring
(C) A wallet
(D) A watch
78. What problem does the speaker ment
82
(A) An additional item was delivered.
(B) An item broke during shipment.
(C) A receipt was not provided.
(D) A discount was not applied.
79. What is the speaker requesting?
(A) Overnight delivery
(B) A refund
(C) A replacement
(D) A price reduction
______________________________
80. What will happen at the bookstore tonight?
(A) A contest winner will be announced.
(B) An author will give a presentation.
(C) A new publication will be released.
(D) A book club will hold a meeting.
81. What is Grace Bradley's book about?
(A) Her photography
(B) Her work with children
(C) Her recent trips
(D) The publishing industry
82. What are listeners invited to do after the event?
(A) Have refreshments
(B) Meet the judges
(C) Register for a membership
(D) Have books signed
______________________________
83. What is the speaker calling about?
(A) A musical instrument
(B) A newspaper subscription
(C) A kitchen appliance
(D) A stereo system
84. What surprised the speaker?
(A) The return policy
(B) The quantity available
(C) The price
(D) The terms of a promotion
85. What would the speaker like to do?
(A) Receive a product brochure
(B) Inspect an item
(C) Place an order
(D) Speak to a manager
______________________________
86. What is the announcement mainly about?
(A) A job opening
(B) A new contract
(C) An employee's achievement
(D) An anniversary celebration
83
87. Who is Juan Mendez?
(A) A member of the board of directors
(B) A client of the company
(C) A professional athlete
(D) A graphic designer
88. What has the company recently changed?
(A) Its business hours
(B) Its logo
(C) Its manufacturing methods
(D) Its prices
______________________________
89. What will visitors learn about on the tour?
(A) The development of an organization
(B) The life of a famous artist
(C) The architecture of a building
(D) The history of an art form
90. Who should make a reservation?
(A) Weekday visitors
(B) Art students
(C) Large groups
(D) Families with young children
91. According to the message, what can listeners do on the Web site?
(A) Register for a workshop
(B) Learn about membership benefits
(C) Volunteer to give tours
(D) View a list of future events
______________________________
92. What will change at the company?
(A) A second factory will open.
(B) A new product will be manufactured.
(C) New equipment will be installed.
(D) Hours of operation will be extended.
93. According to the speaker, why should listeners talk to a supervisor?
(A) To request additional work
(B) To find out about a training session
(C) To suggest improvements
(D) To discuss vacation time
94. When will the change be made?
(A) At the beginning of the week
(B) In two weeks
(C) Next month
(D) Next year
______________________________
95. What event is being described?
(A) A computer programming class
(B) A new-employee orientation
(C) A career-counseling workshop
84
(D) A writing competition
96. What will attendees have the chance to do?
(A) Practice a skill
(B) Participate in a group discussion
(C) Test a new product
(D) Subscribe to an industry publication
97. What will all attendees receive?
(A) An instructional DVD
(B) A reference guide
(C) A certificate of completion
(D) A meal voucher
______________________________
98. What type of product is being discussed?
(A) A desktop computer
(B) A video camera
(C) A mobile phone
(D) A software program
99. According to the report, what is the most notable feature of the product?
(A) It is easy to use.
(B) It is lightweight.
(C) It has a low price.
(D) It has a long battery life.
100. How does the company plan to create interest in the new product?
(A) By rewarding customers who shop early
(B) By hosting in-store demonstrations
(C) By hiring a celebrity spokesperson
(D) By running Internet advertisements

PRACTICE 2

PART 4
Directions: You will hear some short talks given by a single speaker. You will be asked to
answer three questions about what the speaker says in each short talk. Select the best
response to each question and mark the letter (A), (B), (C), or (D) on your answer sheet.
The talks will not be printed in your test book and will be spoken only one time.

71. What does the speaker say about Dr. Lopez?


(A) She has changed her office hours.
(B) She is away on vacation.
(C) She is not accepting new patients.
(D) She is working at a different location.
72. What should callers do to make an appointment?
(A) Hold for an operator
(B) Leave a message
(C) Dial an extension
(D) Call back later in the day
85
73. What will happen on Monday?
(A) Patient calls will be returned.
(B) A medical conference will take place.
(C) A new phone system will be installed.
(D) The office will be closed.
______________________________
74. What is being advertised?
(A) A cleaning company
(B) A computer store
(C) An Internet provider
(D) An accounting firm
75. What does the speaker emphasize about the service?
(A) Its reliability
(B) Its low prices
(C) Its speed
(D) Its ease of use
76. According to the speaker, what is available on the Web site?
(A) Promotional codes
(B) Instructional videos
(C) Software applications
(D) Customer reviews
______________________________
77. Where does the speaker work?
(A) At a dry cleaners
(B) At a bank
(C) At a hotel
(D) At a clothing shop
78. What was found?
(A) A set of keys
(B) A digital camera
(C) A mobile phone
(D) A credit card
79. What will the listener have to show to up the item?
(A) A hotel receipt
(B) A claim check
(C) A business card
(D) A passport
______________________________
80. What is causing the delay?
(A) The crew is stuck in traffic.
(B) The plane has not arrived.
(C) Weather conditions are poor.
(D) A gate is not available.
81. According to the speaker, when will the flight depart?
(A) In 30 minutes
(B) In one hour
(C) In two hours
86
(D) In three hours
82. What is offered to the passengers?
(A) Free headphones
(B) A travel guide
(C) A meal voucher
(D) A discount on future travel
______________________________
83. Who most likely is the speaker?
(A) A musician
(B) A museum guide
(C) A radio host
(D) A film producer
84. According to the speaker, what will happen this summer?
(A) An award will be presented.
(B) A performer will go on tour.
(C) An album will be recorded.
(D) A cultural center will open.
85. What will the speaker do next?
(A) Play some music
(B) Announce an event schedule
(C) Thank some sponsors
(D) Give a lecture
______________________________
86. What does Ludlow Enterprises produce?
(A) Television shows
(B) An electronics magazine
(C) Sports gear
(D) Video games
87. What are listeners invited to do?
(A) Nominate candidates
(B) Submit ideas
(C) Test products
(D) Write reviews
88. According to the speaker, what prize will be awarded?
(A) A free trip to a trade show
(B) Dinner with a celebrity
(C) Electronic equipment
(D) An appearance on television
______________________________
89. What will take place this weekend?
(A) A holiday parade
(B) An automobile show
(C) A sporting event
(D) An arts festival
90. What are drivers asked to do?
(A) Observe speed restrictions
(B) Use alternate routes
87
(C) Avoid parking on streets
(D) Allow extra travel time
91. What will be available for free?
(A) City maps
(B) Event tickets
(C) Bus service
(D) Neighborhood tours
______________________________
92. What is Aviva Rozen's position?
(A) Software executive
(B) Financial advisor
(C) Marketing director
(D) Chief scientist
93. According to the speaker, what did Ms.
Rozen accomplish?
(A) She negotiated a complicated merger.
(B) She implemented a global sales strategy.
(C) She developed the company's training plan.
(D) She started a successful division.
94. What will Ms. Rozen do after she retires?
(A) Volunteer at a museum
(B) Start a new business
(C) Pursue a hobby
(D) Consult for the company
______________________________
95. Why did people gather at the Keller Center this morning?
(A) To visit a plant exhibit
(B) To attend an outdoor concert
(C) To take a gardening class
(D) To listen to an art lecture
96. What is said about the new building?
(A) It is surrounded by water.
(B) It uses solar energy.
(C) It has a rooftop cafe.
(D) It has a sculpture garden.
97. Why was Sean Briggs chosen?
(A) He submitted the lowest bid.
(B) He presented the best designs.
(C) He can start immediately.
(D) He is a local resident.
______________________________
98. Who most likely are the listeners?
(A) Fundraising professionals
(B) Library directors
(C) History professors
(D) Student volunteers
99. What will listeners be doing?
88
(A) Contacting library patrons
(B) Conducting research
(C) Organizing a collection
(D) Coordinating an annual event
100. What will be the subject of Dr. Kim's talk?
(A) How to engage an audience
(B) How to recruit experienced personnel
(C) How to protect documents from damage
(D) How to communicate with donors

89
PRACTICE 3

PART 4
Directions: You will hear some short talks given by a single speaker. You will be asked to
answer three questions about what the speaker says in each short talk. Select the best
response to each question and mark the letter (A), (B), (C), or (D) on your answer sheet.
The talks will not be printed in your test book and will be spoken only one time.

71. When is this broadcast being heard?


(A) In the morning
(B) During the lunch hour
(C) In the late afternoon
(D) In the evening
72. What has caused a problem?
(A) A broken traffic signal
(B) Fallen trees
(C) A community parade
(D) Bridge construction
73. What does the speaker suggest?
(A) Following an alternate route
(B) Using public transportation
(C) Driving carefully
(D) Leaving early
______________________________
74. Where is the museum?
(A) In a renovated factory
(B) In a historic home
(C) On a ship
(D) On a farm
75. What should listeners do if they want to take a guided tour?
(A) Speak with the information desk
(B) Meet at the entrance on the hour
(C) Purchase tickets online
(D) Register in the gift shop
76. Why will part of the museum be closed temporarily?
(A) To host a private party
(B) To set up an exhibit
(C) To install video cameras
(D) To replace some windows
______________________________
77. What event is being rescheduled?
(A) A sales conference
(B) A department seminar
(C) A business luncheon
(D) A retirement party
78. Why was the event postponed?
90
(A) A budget had not been approved yet.
(B) A restaurant was already booked.
(C) A speaker canceled unexpectedly.
(D) A client project had to be finished.
79. What are listeners asked to help select?
(A) A menu
(B) A location
(C) A date
(D) A guest list
______________________________
80. What type of business is being advertised?
(A) An interior design firm
(B) An art supply store
(C) A real estate agency
(D) A home improvement store
81. What is the topic of this Saturday's workshop?
(A) Choosing a decorating style
(B) Installing carpet
(C) Remodeling a bathroom
(D) Framing artwork
82. According to the speaker, what will be distributed at the workshop?
(A) Information packets
(B) Coupons
(C) Supply kits
(D) Color samples
______________________________
83. Who is the speaker?
(A) An event planner
(B) A hotel manager
(C) A landscaper
(D) A photographer
84. What is the problem?
(A) Rain is predicted.
(B) A guest has not arrived.
(C) Some lights are not working.
(D) Some flowers have not been planted.
85. What is planned for this weekend?
(A) A wedding ceremony
(B) A sculpture installation
(C) A photo exhibition
(D) A garden show
______________________________
86. What type of business does the speaker own?
(A) A fitness center
(B) A public relations firm
(C) A food store
(D) A bookshop
91
87. What did Stacy Weston write?
(A) A cookbook
(B) A newspaper article
(C) A restaurant guide
(D) An autobiography
88. What is scheduled for tomorrow morning?
(A) A book signing
(B) A grand opening
(C) A race
(D) A demonstration
______________________________
89. What will be built in the community?
(A) A hospital
(B) A community center
(C) A park
(D) A transit station
90. What benefit of the project did the governor mention?
(A) Shorter commutes
(B) Increased tourism
(C) A cleaner environment
(D) More local jobs
91. Who will be interviewed after the break?
(A) The governor
(B) Community residents
(C) A building inspector
(D) An economics professor
______________________________
92. What product is being discussed?
(A) A stereo system
(B) A laptop computer
(C) A color printer
(D) A software program
93. What is said about Koji Aoki?
(A) He works for an industry magazine.
(B) He created an influential advertising campaign.
(C) He led a successful project team.
(D) He is on the board of directors.
94. What does the company plan to do next year?
(A) Hire a consultant
(B) Open an international office
(C) Change its logo
______________________________
95. In what department does the speaker work?
(A) Research
(B) Payroll
(C) Sales
(D) Personnel
92
96. What is the listener asked to sign on Monday?
(A) An employment contract
(B) A paycheck
(C) An application form
(D) A purchase order
97. Why is Dan Bates leading a session on Monday?
(A) It was the only time that he was available.
(B) He was recently promoted.
(C) The usual presenter will be away.
(D) The listener requested a change.
______________________________
98. What is the subject of the announcement?
(A) A construction project
(B) A charitable donation
(C) A city landmark
(D) A corporate merger
99. According to the speaker, what is a goal of the Penndale Civic Association?
(A) To sponsor community events
(B) To improve the business district
(C) To restore historic neighborhoods
(D) To support local schools
100. What will the speaker most likely do next?
(A) Hand out brochures
(B) Answer questions
(C) Review a timeline
(D) Show a video

READING TEST
_________________________________

In the second section of the TOEIC test, you will be tested on how well you
understand written English. There are three parts to this section with special directions
for each part:
Part 5 Incomplete sentences
Part 6 Text completion
Part 7 Reading comprehension

PART 5: INCOMPLETE SENTENCES (Hoàn thành câu)

93
READING TEST
In the Reading test, you will read a variety of texts answer several different types of reading
comprehension questions. The entire Reading test will last 75 minutes. There are three
parts, and directions are given for each part. You are encouraged to answer as many
questions as possible within the time allowed.

You must mark your answers on the separate answer sheet. Do not write your answers in
your text book.

PART 5
Directions: A word or phrase is missing in each of the sentences below. Four answer
choices are given below and each sentence. Select the best answer to complete the sentence.
Then mark the letter (A), (B), (C), or (D) on your answer sheet.

94
STRATEGY OVERVIEW

LANGUAGE STRATEGIES
You will learn the most common types of grammar items found in Part 5.
 Word families (từ trong nhóm gia đình)
 Similar words (các từ giống nhau)
 Pronouns (đại từ)
 Prepositions (giới từ)
 Coordinate conjunctions (liên từ kết hợp)
 Subordinate conjunctions (liên từ phụ thuộc)
 Verb tense (thì của động từ)
 Phrasal verbs (cụm động từ)

TEST STRATEGIES
 Use grammar clues to figure out the part of speech needed to complete the
statement.
 Read to find the context of the statement. Decide which vocabulary word best fits
the context.
 Find the noun that a pronoun refers to. It is the subject or object of the sentence?
 Identify the position of an item to choose the correct preposition.
 Choose the best conjunction to joint two parts of a statement by deciding whether
the two parts (1) are choices, (2) contradict each other, (3) give similar
information, (4) describe a reason, or (5) give information about time.
 Look for a time expression to choose the verb tense needed.
 Eliminate answer choices that you know are incorrect.
 Move on to the next question if you don’t immediately know the answer.

WORD FAMILY
Word families are words that have the same root but different endings. The ending shows
the part of speech of the word. The ending tells you how a word is used in a sentence. The
root word is often a noun or verb, but it can be any part of speech.

Look at the roots and endings in the chart.


Part of Speech
Root (gốc từ) Noun Verb Adjective Adverb
public publicity publicize (public) publicly
(public)
nation nationality nationalize national nationally
(nation)
95
care (care) (care) careful / carefully/
careless carelessly
wide width widen (wide) widely
accept acceptance (accept) acceptable acceptably
agree agreement (agree) agreeable agreeably

SUFFIXES
 Adverb (Adv): _ly
 Exception: friendly (Adj); manly; monthly (Adv/Adj),…
 Adjective (Adj): _ic; _able; _al; _ful/_less;….
E.g.: energetic; portable; disable; national; careful/careless; beautiful….
 Verb (V): _ize; _en
E.g.: publicize; widen; broaden; lengthen;….
 Noun (N): _ty; _th; _ce; _ment; _tion; _or/er; _ness
E.g.: beauty; breath; width; importance; nation; function; sailor; singer;
thoughtfulness; laziness;…

Sentences
Noun The new product needs a lot of publicity.
Verb Companies publicize products on the Internet, on TV, and in magazines.
Adjective The company made a public announcement about the new product.
Adverb Company representatives announced the new product publicly.

Practice
DIRECTIONS: Mark the choice that best completes the sentence.
1. We will ___V0____ our decision when we sign the paper.

** Modal Verb (động từ khiếm khuyết: will/would; can/could; may/might; have to; ought
to;….) + V0 (bare infinitive: nguyên mẫu)

V0 (nguyên mẫu) V1 (chia động từ ở V2 (chia động từ ở QK) V3 (chia động từ


hiện tại) theo cấu trúc)
Be Am / is / are Was (singular: số ít) Been
Were (plural: số nhiều)
Have Has (singular: số ít) Had Had
Have (plural: số nhiều)

(A) formalize (V)


(B) formal (Adj)
(C) formally (Adv)
(D) formality (N)

2. The ___N____ of your speech cannot be more than 30 minutes.

** Article: A / an / the + Noun (N)


96
(A) long (Adj)
(B) length (N)
(C) lengthen (V)
(D) lengthened (V_ed)

3. The __Adj_____ location for our new office is downtown.


N

Nguyên tắc bổ nghĩa: có 2 thành phần bổ nghĩa gồm ADJ và ADV


** Adj: bổ nghĩa cho N, và đứng trước N
E.g.: This is a beautiful house.
Adj N
He is an intelligent boy.
Adj N

** Adv: bổ nghĩa cho các thành phần (còn lại ngoại trừ N) gồm: V, Adj, Adv, và cả câu.
=> Để biết Adv bổ nghĩa cho thành phần nào, ta chỉ cần đặt câu hỏi với “How?”
E.g.: He wakes up early every day.
Vphrase Adv
She is a very lovely girl.
Adv Adj N
I love you very much.
V Adv Adv

Recently, he has been very busy.


Adv Adj
(A) preference (N)
(B) preferably (Adv)
(C) preferable (Adj)
(D) prefer (V)

4. As soon as everybody is happy with the ___N____, we will sign the papers.
(A) agree (V)
(B) agreeable (Adj)
(C) agreement (N)
(D) agreeably (Adv)

5. It was very ___Adj____ of the boss to buy us those nice gifts. (=> cấu trúc chủ từ giả =>
nhấn mạnh)

It + Vtobe + (Adv) + ADJ + (for O) + to infinitive (To V0) …….


E.g.: It is (very) difficult (for me) to complete my homework by 5 today.
97
(A) thoughtfully (Adv)
(B) thoughtful (Adj)
(C) thoughts (N: plural)
(D) thought (N: singular) => V: think

98
SIMILAR WORDS
Words may look similar, but they may have very different meanings. Sentence context and
grammar will help you decide which is the correct word.

These are some examples of words that look similar:

object liver omit contract


reject liven admit contrast
project livid remit control
deject lively permit contrive

Practice
DIRECTIONS: Mark the choice that best completes the sentence.
1. Everyone was ___V3/ed____ with the high quality of Mr. Clark’s work.

** Passive Voice (Bị động): BE + V3/ed => bị / được


(A) compressed
(B) oppressed
(C) impressed
(D) repressed

2. We will hire a legal ___N____ to prepare the documents.


Adj
(A) expect (V) – N: expectation
(B) exploit (N/V)
(C) export (N/V)
(D) expert (N)

3. Your ___N____ during our visit was greatly appreciated.


Poss. Adj. (tính từ sở hữu)
(A) hospice (N: việc chăm sóc đặc biệt)
(B) hospital (N: bệnh viện)
(C) hospitality (N: lòng hiếu khách)
(D) hospitalize (V)

4. You will need to ___V0____ the check before we can cash it.
(A) sing (V: ca hát)
(B) sign (V: ký tên)
(C) sigh (V: thở dài)
(D) sight (V: ngắm cảnh / xem)

5. The new employee ___V____ to us that he didn’t have much experience.


S
99
(A) omitted
(B) remitted
(C) permitted
(D) admitted (thừa nhận)

100
PRONOUNS
Pronouns take the place of nouns. Subject pronouns take the place of the noun object of a
sentence or clause. Subjects come before a verb. Object pronouns come after a verb or
preposition.
Subject Pronouns Object Pronouns
Singular Plural Singular Plural
I we me us
You you you you
he, she, it they him, her, it them

Pronouns Object Possessive Possessive Reflexive Pronouns


Pro. Adjective (+N) Pronouns (nhấn mạnh)
I I: Singular Me My Mine Myself
We I: Plural Us Our Ours Ourselves
You II: Sing./plu. You Your Yours Yourself/yourselves
He III: Singular Him His His Himself
She Her Her Hers Herself
It It Its Its Itself
They III: Plural Them Their Theirs Themselves

** Singular: _self
** Plural: _selves

** SS giữa Poss. Adj. (tính từ sở hữu) và Poss. Pro. (đại từ sỡ hữu)


E.g.: This is my lovely house.
This is a lovely house. It is mine.

** Đại từ phản thân (Reflexive pronouns) => dùng để nhấn mạnh / làm rõ vai trò của chủ
thể
E.g.: I went to school by myself.
I myself went to school.
I myself did the homework.
 Đại từ phản thân có thể đứng cuối câu hoặc liền ngay sau chủ từ. Nếu đại từ phản
thân đứng cuối câu phải có giới từ “by”.

Sentences
Mr. and Mrs. Brown are at the hotel now. They arrive by plane last night.
We bought a new computer. It is on the desk in my office.
George and I are going to the conference tomorrow. My boss will wait for us at 6:00.
Mr. Lang knows the answer. You should ask him.

Practice
DIRECTIONS: Mark the choice that best completes the sentence.
101
1. The door is open. Shut ___O____ if you want.
(A) them
(B) him
(C) her
(D) it

2. I’ll finish this job faster if I can find someone to work with ___O____.
(A) you (x)
(B) we (x)
(C) him (x)
(D) me

3. Mr. Yeoh is our computer expert. ___S/pro____ will help you with that problem.
(A) He
(B) She
(C) Him (O) => x
(D) Her (O) => x

4. The manager gave us a day off because ___S/pro____ worked extra hours last week.
(A) he
(B) we
(C) us (O) => x
(D) him (O) => x

5. My brother’s wife is an architect. ___S/pro____ designed my house.


(A) He
(B) She
(C) You
(D) It

PREPOSITIONS
Prepositions come before nouns. Prepositions can show the location of an object. Often this
is in relation to another object. Prepositions can also show the direction of a moving object.

Placement (vị trí) Direction (phương hướng)


+ behind / back: đằng sau / phía sau
+ in front of: trước mặt
+ across from / opposite to: đối diện
+ at / in: I’m waiting for you at the school.
I’m waiting for you in the school.
in on to from

102
around

over/above under/ below/


beneath

through (xuyên suốt)

next to/ beside/ by

103
Practice
DIRECTIONS: Mark the choice that best completes the sentence.
1. He carried all those heavy packages _______ the store to the office.
(A) to
(B) from (….from….to….: từ đâu đến đâu)
(C) under
(D) in

2. The mail carrier left the mail _______ your desk.


(A) at
(B) above
(C) to
(D) on

3. You have to walk _______ the lobby in order to get to the elevators.
(A) over
(B) in
(C) through
(D) on

4. They decided to rent an office _______ a large building downtown.


(A) in
(B) above
(C) to
(D) on

5. We will have better light if we sit _______ the window.


(A) from
(B) to
(C) around
(D) beside (by / next to)

104
(Liên từ kết hợp)
COORDINATE CONJUNCTIONS
Conjunctions joins words, phrases, and clauses. Coordinate conjunctions join two or more
equal grammar structures: two nouns (N), two verbs (V), two verb phrases (VP), two
independent clauses (IC), etc.
** FANBOYS
For = purpose (chỉ mục đích)
And = adds similar information (cung cấp thêm thông tin có giá trị tương đương)
Nor = shows a negative choice (miêu tả sự phủ định)
But = shows a contradiction (miêu tả sự đối lập)
Or = shows a choice (thể hiện sự lựa chọn)
Yet = shows a reason (thể hiện nguyên nhân)
So = shows a result (thể hiện kết quả)

** Correlative Conjunctions (liên từ tương hỗ)


+ ….not only ….but (also)…..:……không những…mà còn ….
+ ….both ….and….: cả 2
+ Either….or….: cái này hoặc cái kia
+ Neither ….nor….: cả 2 đều không

Conjunction Use
and adds similar information
but shows a contradiction
or shows a choice
nor shows a negative choice

Sentences
N+N Neither my boss nor I will be at the office next week.
V+V We can stay for the lecture or leave now.
VP + VP I will finish the report and submit it before Friday.
IC + IC He works long hours, but he doesn’t make much money.

Practice
DIRECTIONS: Mark the choice that best completes the sentence.
1. We need to order some computer paper _______ several boxes of envelopes.
(A) or
(B) but
(C) either => x
(D) and

2. Neither Ms. Chen _______ Mr. Martinez was able to attend the seminar.
(A) nor
(B) and
(C) neither
105
(D) or

3. We can sit (either) by the window _______ next to the door.


(A) or
(B) and
(C) nor
(D) but

4. He went to the train station _______ bought a ticket to New York.


(A) but
(B) and
(C) nor
(D) or

5. You can look at the clothes in the shop windows, _______ you can’t buy any of them.
(A) but
(B) or
(C) nor
(D) neither

(liên từ phụ thuộc)


SUBORDINATE CONJUNCTIONS
Subordinate conjunctions join two clauses: an independent clause (IC) and a dependent
clause (DC). An independent clause can stand alone; a dependent clause cannot. The
conjunction introduces the dependent clause. The dependent clause can come before the
independent clause or after it.

Conjunction Use
because introduces a reason
since
although introduces a contradiction
though
even though
before introduces a time clause
after
when
whenever
while
as soon as

Sentences
IC + DC The office closed early because the weather was so bad.
DC + IC Even though they worked all night, they weren’t able to meet the deadline.
IC + DC He checked his messages as soon as he arrived at the office.

106
Practice
DIRECTIONS: Mark the choice that best completes the sentence.
1. ___Conj____ they were tired, they worked overtime.
(A) Because (bởi vì)
(B) Whenever (mỗi khi)
(C) Since (từ khi)
(D) Although (mặc dù)

** Although + Clause (S + V + (O)…), but …….


E.g.: Although I got up early, I went to school late.

2. Mr. Kim visits our office ___Conj____ he is in the neighborhood.


(A) before (trước khi)
(B) but (nhưng)
(C) whenever (mỗi khi)
(D) even though (mặc dù = although / though)

3. ___Conj____ Mr. Park worked for us, he had received training abroad.
(A) Before (trước khi)
(B) As soon as (ngay khi)
(C) Either (…or..)
(D) Whenever (mỗi khi)

4. Please answer the phone ___Conj____ it rings.


(A) even though (mặc dù)
(B) when (khi)
(C) and (và)
(D) before (trước khi)

5. He took a second job _______ he needed extra money.


(A) because
(B) though
(C) before
(D) but

VERB TENSE
The main verb tells you when the action in the sentence occurs. Look for time expressions.
Certain time expressions indicate certain verb tenses. The verb tense and the time
expressions often appear together.
Verb tense Use Time expressions
Simple Present Habit every: every day, every week,
every month
adverbs of frequency: always,
107
usually, often, sometimes, never,
rarely
Present Continuous An action occurring now now
at this moment
at this time
currently
Simple Past An action completed in the last: last night, last weekend, last
past june, last year
ago: an hour ago, two weeks ago
Present Perfect An action started in the since: since 1981, since last
Present Perfect Continuous past and continuing to the night
present for: for two month, for a long
time
Future An action occurring in the next: next monday, next
future summer, next may
tomorrow: tomorrow morning,
tomorrow evening

Types Original Past Present Future


Structure => V V => V2 V => V1 Will => V0
Simple V V2/ed V: Splural Will + V0
(đơn) Vs/es: Ssingular
Continuous Be + V_ing Was/were + V_ing Am/is/are + V_ing Will + be + V_ing
(tiếp diễn) (đang) (đang) (đang) (đang)
Perfect Have + V3/ed Had + V3/ed Has/Have + V3/ed Will + have + V3/ed
(hoàn thành) (đã rồi) (đã rồi) (đã rồi) (đã rồi)
Perfect Cont. Have + been + Had + been + Has/Have + been + Will + have + been +
(hoàn thành tiếp diễn) V_ing V_ing V_ing V_ing

108
Practice
DIRECTIONS: Mark the choice that best completes the sentence.
1. A messenger _______ the package tomorrow afternoon.
(A) delivers
(B) will deliver
(C) has delivered
(D) delivered
2. Ms. Brigham’s assistant _______ here for quite a while.
(A) works
(B) is working
(C) has been working
(D) will work
3. The Paris branch of our bank _______ five years ago today.
(A) opens
(B) has opened
(C) opened
(D) was opening
4. My assistant usually _______ my messages early in the morning.
(A) is checking
(B) has been checking
(C) will check
(D) checks
5. My supervisor promises that I _______ a raise next year.
(A) have gotten
(B) am getting
(C) will get
(D) get

PHRASAL VERBS
Phrasal verbs usually have two parts: a verb and a particle (give back). Some phrasal verbs
have two particles (give up on). The particles look like prepositions, but they are part of the
verb. The meaning of a phrasal verb is not usually related to the meaning of the parts. There
are no rules to help you predict the meaning. You have to learn what each one means. Some
have more than one meaning.

Verb Meaning
turn on start a machine
turn off stop a machine
turn in submit
turn up appear

109
Practice
DIRECTIONS: Mark the choice that best completes the sentence.
1. I would like to call this client, so please _______ his phone number in the directory for
me.
(A) look to
(B) look up
(C) look on
(D) look out
2. Who will _______ the manager’s projects while she is on vacation?
(A) take away
(B) take up
(C) take over
(D) take off
3. The personnel officer believes that we can _______ with our present staff.
(A) get off
(B) get on
(C) get up
(D) get by
4. Don’t forget to _______ the photocopier after you have made your copies.
(A) turn on
(B) turn in
(C) turn up
(D) turn off
5. If I don’t answer the phone, leave a message and I will _______ right away.
(A) call up
(B) call back
(C) call on
(D) call off

110
PART 5 PRACTICE
PART 5
Directions: A word or phrase is missing in each of the sentences below. Four answer
choices are given below and each sentence. Select the best answer to complete the
sentence. Then mark the letter (A), (B), (C), or (D) on your answer sheet.

101. Mr. Vernon would like to _____ his account with our community bank, as he is moving to
another country and needs to change to an international bank.
(A) open
(B) withdraw
(C) deposit
(D) close

102. _____________, we are occupied with preparing for our end-of-year evaluations.
(A) Shortly
(B) Currently
(C) Equally
(D) Quickly

103. The diner around the corner _____ burgers, sandwiches, and coffee.
(A) eats
(B) creates
(C) gives
(D) Serves

104. _____ honor of Mr. Reynolds’ lifetime of commitment and service to the community, a
banquet will be held this Saturday at the Civic Center.
(A) Under
(B) At
(C) In
(D) Of

105. _____ product improvements have significantly increased consumer satisfaction as well as
sales.
(A) Continuous
(B) Prosperous
(C) Mature
(D) Straight

106. The sign says _____ all deliveries must be brought to the back door.
(A) which
(B) that
(C) where
(D) who
107. To turn this country around, politicians need to take much _____ steps to reform tax and
other economic policies.
(A) boldness
(B) bolder
(C) boldest
(D) boldly

108. _________ Tuesday, all employees are required to wear a tie during working hours.
111
(A) Outside
(B) Starting on
(C) Afterward
(D) Instead of

109. Stephen is looking to buy a new house, ________ his first house still hasn’t sold.
(A) if
(B) when
(C) even
(D) although

110. The city commissioner that will be instead next year has released a report indicating city
_____ practices.
(A) development
(B) developing
(C) developed
(D) develops

111. The company founder continues to show _____ to change and will most likely be asked to
resign in the near future.
(A) resisting
(B) resistant
(C) resistance
(D) resisted

112. If you are interested in the Director of Music and Visual Arts position, please _____ your
résumé and cover letter.
(A) comply
(B) submit
(C) urge
(D) advice

113. All articles for the monthly newsletter must be received _____ the 15th of the month.
(A) at
(B) by
(C) between
(D) except

114. One of the office secretaries will attend all meetings and _____ the items discussed and
decisions made.
(A) record
(B) practice
(C) substitute
(D) Criticize

115. Once you fill out the form and complete the questionnaire, please submit _____ to Mindy
Carlson, my assistant.
(A) they
(B) their
(C) them
(D) theirs

112
116. There is a three-car crash on Freeway 670, _____ drivers are advised to take alternative
routes.
(A) except
(B) if
(C) so
(D) because

117. Sales of diamond rings _____ increase during this time of the year.
(A) openly
(B) rigidly
(C) approximately
(D) frequently

118. Run Community Bank will not be open on _____ New Year Eve’s or New Year’s Day, due
to the holiday.
(A) either
(B) neither
(C) both
(D) with

119. Since Mrs. Redcliff was not happy with the return on her stock investment, she has _____ her
account to a financial planner.
(A) spent
(B) closed
(C) overdrawn
(D) transferred

120. It is predicted that the demand for hybrid vehicles _____ exponentially over the next decade.
(A) be grown
(B) growing
(C) has grown
(D) will grow

121. The mechanic told Tom that he needs to buy a new transmission for _____ car.
(A) he
(B) himself
(C) his
(D) him

122. This job requires applicants to be willing to travel _____ to the company’s offices around the
nation.
(A) regular
(B) regularly
(C) regularity
(D) regulate

123. We would like to thank all the families for their _____ in this fundraising event.
(A) participate
(B) participated

113
(C) participating
(D) participation

124. In order to lead a fulfilled life, you need to have not only a satisfying job but _____ a healthy
social life.
(A) altogether
(B) also
(C) alone
(D) almost

125. All employees must sign out at the front desk _____ leaving the facility for the evening.
(A) before
(B) until
(C) because
(D) then

126. _____ among Internet providers is expected to increase dramatically over the next year.
(A) Competition
(B) Competitor
(C) Competitive
(D) Compete

127. The city has received federal funding to _____ the historical City Hall building to its former
beauty.
(A) restart
(B) regain
(C) restore
(D) replace

128. The graduation speaker congratulated the graduating class on working _____ hard these past
four years.
(A) exceptionally
(B) exception
(C) exceptional
(D) except

129. The temporary position is open now _____ the end of March.
(A) since
(B) through
(C) between
(D) while

130. All lottery money will be divided _____ those holding winning tickets.
(A) toward
(B) among
(C) after
(D) during

131. Some people just cannot _____ a living on a minimum wage.


(A) get
(B) survive
(C) make
(D) do

132. There are certain equipment repairs that can only be made by a trained_____
114
(A) technician
(B) factory
(C) certificate
(D) generator

133. All passengers must _____ a valid form of identification before entering the aircraft.
(A) notify
(B) assign
(C) permit
(D) present

134. In order to _____ funds, you must show two forms of identification.
(A) withdraw
(B) earn
(C) cost
(D) spent

135. The store will be able to open its doors for business_____ the city official has inspected the
building and made sure it passes all codes.
(A) nevertheless
(B) despite
(C) still
(D) once

136. _____ you choose to continue our professional relationship, we would like to extend to you a
20 percent discount for your troubles.
(A) Whereas
(B) As if
(C) Whether
(D) Assuming that

137. Timothy’s supervisor commented on how_____ he has been with the initiative which
Timothy has shown during his first year at the company.
(A) impression
(B) impressed
(C) impress
(D) impresses

138. The decline in product sales is _____ due to market saturation, which should have been
predicted and planned for.
(A) probable
(B) probability
(C) probably
(D) probabilities

139. To reach an _____ temperature, we recently have installed an intelligent sensor in the living
room.
(A) optimal
(B) optimize
(C) optimally
(D) optimizing

140. All questions _____ the items on the meeting agenda should be directed to Sam Jones, Head
of Human Resources.
(A) relating
115
(B) connecting
(C) concerning
(D) referring

141. A good résumé should highlight your best_____, making you a desirable applicant for the
job.
(A) qualifications
(B) qualified
(C) qualifying
(D) qualifies

142. The art _____ at City Museum has been just extended until next month.
(A) exhibition
(B) guide
(C) portrait
(D) creativity

143. Sam’s manager asked him to _____ in the conference call regarding the past quarter’s
performance statistics.
(A) complete
(B) attend
(C) participate
(D) release

144. Residents of Joyville always help visitors _____ historical sites or other destinations in their
beloved city.
(A) assist
(B) remind
(C) locate
(D) conduct

145. The proprietor of Fast Cash Loan Services was excited to open _____ store’s doors for the
first time.
(A) he
(B) his
(C) him
(D) himself

146. Senior management is gathering now to discuss how to run the company more _____.
(A) efficient
(B) efficiency
(C) efficiencies
(D) efficiently

147. During the full-day seminar, your necessary materials and lunch will _____ by the event
organizer.
(A) be provided
(B) provide
(C) to provide
(D) be providing

148. Mark evaluated his monthly _____ to see if he could afford to take a vacation to the
Bahamas.
(A) expenses
116
(B) resources
(C) credits
(D) prices

149. Harold Dennis Co., a high-end motorcycle manufacturer, predicted there would be a strong
_____ for its newest model HD-365 in Asia within the next few days.
(A) emission
(B) market
(C) revision
(D) purchase

150. The compliance department will _____ audits on various business units on a regular basis.
(A) conduct
(B) visit
(C) inspect
(D) arrive

151. I intend for the _____ to the banquet to be sent out by the end of the week.
(A) invitations
(B) invite
(C) inviting
(D) invitation

152. Mr. Yamamoto has given Ms. Apelman his _____ that all the documents will be ready by
the first week of April.
(A) assurance
(B) assuredly
(C) assured
(D) assure

153. Miranda is admired for her _____, as she is able to take things as they come and change
focus when needed.
(A) response
(B) collection
(C) experiments
(D) flexibility

154. The restaurant has the _____ to refuse service to any customers not wearing proper attire.
(A) importance
(B) selection
(C) right
(D) goal

155. The municipal health bureau reminded the residents to apply _____ to get a seasonal flu
shot on time.
(A) usefully
(B) early
(C) enough
(D) hardly

156. When we were living in Seattle, my wife and I used to go dancing _____ other week.
(A) some
(B) several
117
(C) every
(D) most

157. The _____ of our organization are to provide education, training, and resources to
unemployed individuals looking for new employment.
(A) goals
(B) destinations
(C) opinions
(D) treatments

158. Ms. Palin has suggested that we _____ renting the property rather than purchasing it.
(A) considering
(B) considered
(C) consider
(D) are considering

159. If you don’t mind, I would like to close the door, as I have a _____ matter which I need to
discuss with you.
(A) imperative
(B) confidential
(C) secure
(D) notable

160. Only those who are _____ to the state can receive that particular discount at the state
university.
(A) native
(B) according
(C) physical
(D) approximate

161. The seminar _____ with a question-and-answer session in which audience members were
invited to ask questions of the expert panel.
(A) conclusion
(B) concluding
(C) to conclude
(D) concluded

162. He has not been the same man _____ his wife died in a horrible car accident last year.
(A) whether
(B) since
(C) for
(D) which

163. My lunch break is only 30 minutes, so we have to eat somewhere _____ a quick walk from
here.
(A) across
(B) within
(C) in front of
(D) nearby

164. Your Members Only Credit Card may be used _____ major credit cards are accepted.
(A) wherever
118
(B) alongside
(C) together
(D) forward

165. When polled, voters pointed out that they are _____ with the candidate’s name and
background.
(A) profitable
(B) filled
(C) unfamiliar
(D) difficult

166. A law banning new construction in the historic downtown area was passed after nearly a
year of citywide _____ over the issue.
(A) debate
(B) performance
(C) rebate
(D) action

167. Jacques was _____ in seeing if the fashion designer’s new clothing line would include
menswear.
(A) interest
(B) interests
(C) interested
(D) interesting

168. Rumors that company intend to _____ a merger with its competitor have been driving its
stock value through the roof.
(A) announce
(B) agree
(C) involve
(D) maintain

169. In China, it is not _____ to bring up problems right away in conversations.


(A) moral
(B) maligned
(C) polite
(D) patriarchal

170. Before you _____, please complete the conference review form, describing what you found
beneficial and what may need improvement.
(A) to leave
(B) had left
(C) leaving
(D) leave

171. In order to respond __________ to customer complaints, the company management decided
to set up a new team that exclusively deals with complaints over the phone and via email.
(A) overtly
(B) intensely
(C) severely
(D) promptly
119
172. Currently, students are working on their advertising campaign ________ producing a TV
commercial.
(A) for
(B) by
(C) since
(D) until

173. All items must be fastened __________ to the ship prior to its departure from the dock.
(A) more secure
(B) secure
(C) securely
(D) security

174. For more information on Convenient Customer Access, __________our website, and take a
tour.
(A) visiting
(B) visit
(C) is visiting
(D) to visit

175. With the approach of the holiday season, employees are __________ awaiting their bonuses.
(A) anxiety
(B) anxious
(C) anxiousness
(D) anxiously

176. The more we spent with the sales team, the more __________ we were with their innovative
marketing skills.
(A) impression
(B) impress
(C) impresses
(D) impressed

177. Requirements for the open position include basic computer skills and managerial experience
in __________ sales.
(A) retailing
(B) retail
(C) retailer
(D) retailed

178. the patient's condition get worse, the nurse will call in an internist.
(A) Might
(B) Unless
(C) Should
(D) If
120
179. Since the corporate system has a structured daily schedule, it would be nearly
impossible to impair the work efficiency.
(A) hopefully
(B) highly
(C) probably
(D) rarely

180. The industrial park construction for the past three years, had been scheduled
for completion by late spring.
(A) over
(B) in
(C) of
(D) under

181. The Business Advisor/ Council has been specifically designed or those in _______ of
advice about setting up new businesses.
(A) need
(B) want
(C) duty
(D) absence

182. At the current speed of production, Supreme Tech is not able to manufacture _______
products to meet the demands of our purchase orders.
(A) full
(B) quick
(C) enough
(D) overall

183. the flight is canceled, the seminar will have to be postponed.


(A) While
(B) If
(C) Although
(D) Besides

184. It is advisable to keep your operating system and software updated for _________
computer performance and security.
(A) optimal
(B) optimize
(C) optimally
(D) optimization

121
PART 6: TEXT COMPLETION
PART 6
Directions: Read the texts that follow. A word or phrase is missing in some of the
STRATEGY
sentences. Four answer choices OVERVIEW
are given below each of the sentences. Select the best
answer to complete the sentence. Then mark the letter (A), (B), (C), or (D) on your answer
LANGUAGE
sheet. STRATEGIES
You will learn the most common types of grammar items in the text completion passages
in Part 6.
Words in context
Pronouns
Prepositions
Time clauses
Adjective comparison
Gerunds and infinitives

TEST STRATEGIES
 Read the sentences before and after the blank to figure out the context.
 Look for a noun after the blank to tell you whether you need an adjective or a
pronoun.
 Read the sentences before and after the blank to figure out when the action
occurs.
 Look at the verb tense in one part of the sentence to figure out what tense is
needed in the other part of the tense.
 Look for the and than in comparisons to determine the correct adjective form.
 Figure out whether the main verb is followed by a gerund or an infinitive.
 Move on to the next question if you don’t immediately know the answer.

122
PRACTICE
PART 6
Directions: Read the texts that follow. A word or phrase is missing in some of the
sentences. Four answer choices are given below each of the sentences. Select the best
answer to complete the sentence. Then mark the letter (A), (B), (C), or (D) on your answer
sheet.

Questions 185-187 refer to the following letter.

Dear Mr. Sanchez,


My name is Francis Arbuckle. I am the publisher of an information technology magazine called
Today's Computer. We are currently planning on putting out our next _____, which will focus on
the use of robots in modern industry.
185.
(A) show
(B) investment
(C) issue
(D) episode
We _____to interview a successful modern industrial executive such as yourself. If you are
186.
(A) would like
(B) will like
(C) liked
(D) have liked
interested and willing to _____ us over the next couple of weeks, please phone me at 878- 993-
2424.
187.
(A) talk to
(B) present
(C) regulate
(D) wait for
We eagerly await your call.
Regards,
Francis Arbuckle
Publisher
Today's Computer Magazine

123
Questions 188-190 refer to the following article.

Increasing Cost of Food Staples Pressures Families

The cost of milk, eggs and bread is on the minds of every grocery shopper. In the past six months,
the average cost for these items has risen almost 50 percent. For example, _____ with the price tag
188.
(A) contrasting
(B) promising
(C) similar
(D) False
of 98 cents per dozen last May, a dozen eggs now costs an average of $1.50.

And it doesn’t end there. Economists project that these prices will continue _____ throughout the
rest of the year, ending in a total price increase of about 75 percent.
189.
(A) rising
(B) tripling
(C) declining
(D) Decreasing

The _____ of these increasing costs is felt most by the average citizen. A family of four is now
struggling
190.
(A) effect
(B) effecting
(C) effected
(D) effective
to put the most basic food items in their fridge. The money has to come from somewhere, and so
credit card debt is growing while retail shopping is decreasing.

Something must be done before the ripple effect has a dramatic consequence on our economy as a
whole.

124
Questions 191-193 refer to the following announcement.

Attention, managers and human resources specialists:

Rudy Chavez, renowned author and expert on workforce development and management, will
preside over a panel discussion on how to enhance employee _____ without hiring more
191.
(A) product
(B) productive
(C) productivity
(D) productively
resources. The seminar will be held in the Conference Hall at Jupiter Hotel on 11 August at 9:30
a.m.

The _____ of the event is actually adapted from Mr. Chavez’s latest book, Produce More with Less,
192.
(A) content
(B) attendance
(C) contract
(D) invitation
which has received positive reviews both from academics and business managers round the nation.
All the attendees will be eligible to purchase a copy of the book signed by Mr. Chavez himself upon
the conclusion of the event.

The event _____ by the Minneapolis Chamber of Commerce and City Business Development
Bureau.
193.
(A) sponsors
(B) will sponsor
(C) was sponsored
(D) is being sponsored
Do not miss this opportunity. Call us at (613) 645-1179 today.

125
Questions 194-196 refer to the following advertisement.

RWA Looking for Project Coordinator

Roberts, Wayne, and Associates (RWA) is an accounting services firm with a presence in seven
cities in Northern California. We are currently recruiting a project coordinator.

The project coordinator _____ the firm’s accountants and consultants with all aspects of client
service
194.
(A) to assist
(B) are assisting
(C) will be assisting
(D) would have assisted
engagements from planning to collecting fees.

This is a great opportunity for college graduates with project management experience to get into a
professional service business. _____, we are accepting applications from professionals in and
outside of
195.
(A) For instance
(B) Therefore
(C) As a result
(D) However
the accounting industry.

The ideal candidate must be organized and detail-oriented. The candidate must also be _____ in
using standard business software tools such as a spreadsheet and database.
196.
(A) proficient
(B) operational
(C) specific
(D) technical
For the immediate consideration, send your résumé to career@RWACPA.com.

126
Questions 197 - 199 refer to the following letter.

Dear Jordan,
We are pleased to extend to you an offer of employment at our recreation center. We would like
you to work 40 hours a week, and the _______will be $14 an hour.
197.
(A) charge
(B) estimate
(C) cost
(D) pay
Should you choose to accept this offer, please _______ to notify us within 5 business days.
198.
(A) remembering
(B) to remember
(C) remember
(D) are remembered

Your first day of work will be May 2, when you enter our 2-week training course. Upon your
successful completion of the course, you will man the rock wall and weight room in our workout
facility.

All uniforms will be _______, but we do request that you wear your own athletic shoes with rubber-
soled bottoms.
199.
(A) requested
(B) provided
(C) retrieved
(D) informed
We hope to work with you starting May 2.

Sincerely,

Robert Bell
Human Resources, City Recreation Center

127
Questions 200 - 202 refer to the following email.

Date: September 3
To: Sara Moss
From: Ryan Armor
Re: Monday’s Meeting

Dear Sara,

I would like to organize a catered lunch ______ Monday’s staff meeting.


200.
(A) past
(B) between
(C) following
(D) onto

The staff has been working diligently on the new product line. However, I’ve heard that the staff
_____ a
201.
(A) will experience
(B) has been experiencing
(C) will have experienced
(D) had experienced
lack in motivation lately, which is only normal after working countless overtime hours for the past
month, I think a gesture of our appreciation is in order to rejuvenate them. In my experience, food is
the easiest way to say thank you and please everyone at one time. Hopefully, this act will bring
about a(n) ______ in
202.
(A) increase
(B) contribute
(C) prevent
(D) approve
productivity, and we will be done with the project soon.

Please coordinate with the office caterer for Monday’s lunch and let me know if you need my help.

Sincerely,

Ryan
Manager, Soft Tech

128
Questions 203-205 refer to the following announcement.

Attention, managers and human resources specialists:

Rudy Chavez, renowned author and expert on workforce development and management, will
preside over a panel discussion on how to enhance employee _____ without hiring more
203.
(A) product
(B) productive
(C) productivity
(D) productively
resources. The seminar will be held in the Conference Hall at Jupiter Hotel on 11 August at 9:30
a.m.

The _____ of the event is actually adapted from Mr. Chavez’s latest book, Produce More with Less,
204.
(A) content
(B) attendance
(C) contract
(D) invitation
which has received positive reviews both from academics and business managers round the nation.
All the attendees will be eligible to purchase a copy of the book signed by Mr. Chavez himself upon
the conclusion of the event.

The event _____ by the Minneapolis Chamber of Commerce and City Business Development
Bureau.
205.
(A) sponsors
(B) will sponsor
(C) was sponsored
(D) is being sponsored
Do not miss this opportunity. Call us at (613) 645-1179 today.

129
Questions 206-208 refer to the following advertisement.

RWA Looking for Project Coordinator

Roberts, Wayne, and Associates (RWA) is an accounting services firm with a presence in seven
cities in Northern California. We are currently recruiting a project coordinator.
The project coordinator _____ the firm’s accountants and consultants with all aspects of client
service
206.
(A) to assist
(B) are assisting
(C) will be assisting
(D) would have assisted
engagements from planning to collecting fees.
This is a great opportunity for college graduates with project management experience to get into a
professional service business. _____, we are accepting applications from professionals in and
outside of
207.
(A) For instance
(B) Therefore
(C) As a result
(D) However
the accounting industry.
The ideal candidate must be organized and detail-oriented. The candidate must also be _____ in
using standard business software tools such as a spreadsheet and database.
208.
(A) proficient
(B) operational
(C) specific
(D) technical
For the immediate consideration, send your résumé to career@RWACPA.com.

130
Questions 209-211 refer to the following letter.

Mr. Yousef Granger 1736 Houston Ave.


St. Louis, MO 63101

Dear Mr. Granger,

I _______ to inform you that your subscription with us has been canceled because an
209.
(A) am pleased
(B) regret
(C) have faith
(D) am sincere
invoice of $146.50 is now three months overdue. When signing up for the book club, you requested
to be notified in two ways: by text and by email. We have sent four reminders about your invoice in
each of these formats. We _______ this information to your cell phone, and also emailed the
notifications to your account.
210.
(A) dialed
(B) called
(C) posted
(D) texted
_______ this now means that as of November 5, your Books Beyond subscription is no longer
valid.
211.
(A) Surprisingly
(B) Fortunately
(C) Unfortunately
(D) Interestingly

If there has been an error and you have already sent payment, please call 1-800-bks-bynd to clear up
any misunderstanding.

Sincerely,
David Holt
Accounts Specialist

131
Questions 212-214 refer to the following passage.

Over the past 6 months, Fort McKenzie has _____beyond its traditional role as a military
community into a major tourist attraction.
212.
(A) promoted
(B) removed
(C) determined
(D) evolved
Twenty minutes north of Highway 45, Fort McKenzie's renovated historical landmarks are
beginning to attract a lot of attention from tourist groups and students on school trips.
Among its _____ attractions are a historical fort and tours of three military museums.
213.
(A) featured
(B) featuring
(C) features
(D) to features
Because of active participation by new souvenir boutiques and information booths, and offering a
variety of restaurant options for families, the fort is _____ a must-see tourist site.
214.
(A) early
(B) still
(C) soon
(D) now

132
Questions 215-217 refer to the following memorandum.

Tenant Memorandum
January 5. 2007
Dear Tenants.
We will be pressured to wash the parking lot this
Sunday. January 7. 2007. For this reason. The
parking structure will not be (215)
to tenants. We apologize
185.
(A) eligible
(B) capable
(C) available
(D) aware

(216) any inconvenience this may


cause. Thank you for your cooperation.
186.
(A) in
(B) to
(C) for
(D) during

(217) you have any questions regarding


187.
(A) Will
(B) Could
(C) May
(D) Should
this memo, please feel free to call me at 213-380-
2222.

133
Questions 218 - 220 refer to the following
notice.

NOTIC
E

The Crestview branch of Dover Bank has discontinued its services. There are two Dover Bank
branches located near here. One is on the corner of Vernon Street and Fourth Avenue in
the Maple Park area (218) is at the east entrance to the Jensen Shopping Plaza,
which is
215.
(A) Other
(B) The other
(C) Another
(D) Each other

less than a mile from here on Valleydale Boulevard. This change at the Crestview branch is part
of a process of (219) our branches. Our aim is to keep our cost as low as possible so
that
216.
( A ) participating
(B) concluding
(C) attending
(D) consolidating

we can economically provide banking services for the community. We regret that it is
necessary to close this branch because we are (220) of t h e convenience that it
offers
217.
(A) active
(B) aware
(C) careful
(D) awaken

to the immediate neighborhood. However, since there are two other branches close by, we
trust that banking with us will still be quick and easy.

134
PART 7: READING COMPREHENSION
PART 7
Directions: In this part you will read a selection of texts, such as magazine and newspaper
articles, letters, and advertisement. Each text is followed by several questions. Select the
best answer for each question and mark the letter (A), (B), (C), or (D) on your answer
sheet.

STRATEGY OVERVIEW
LANGUAGE STRATEGIES
Be prepared for four question types on the TOEIC test:
Main idea
Detail
Inference
Vocabulary
You will find these questions in both the single passage and double passage section of
Part 7. Look at the samples of each question type.
 MAIN IDEA QUESTIONS
What is being advertised?
What is the purpose of the letter?
What is the main idea of this article?
What is this announcement about?
` What is the purpose of this paragraph?
What is the reason for this correspondence?
What is the topic of the meeting?
What is being discussed?
 DETAIL QUESTIONS
How much is a (product)?
When was the email sent?
What percentage of users are over 30?
Who is (name or title)?
What dates are critical?
Who has to attend the meeting?
How much time does the graph cover?
Where is Mr. Brown working now?

135
 INFERENCE QUESTIONS
Who might use (the product)?
What is the tone of the memo?
Who would use the information?
Who would most likely read this report?
What is the writer’s opinion?
Where would you find these instructions?
Why did Ms. Jones write this letter?
What will the employee do next?
 VOCABULARY QUESTIONS
The word “promotion” in paragraph 1, line 3, is closest in meaning to …
The word “competent” in line 2 is closest in meaning to …
The word “produce” in line 1 of the ad is closest in meaning to …
The word “data” below the graph is closest in meaning to …
 DETAIL QUESTIONS: TIME
When will the meeting take place?
How long will the conference last?
 DETAIL QUESTIONS: REASON
Why will Mr. Wilson have to call the travel agent?
Why does Ms. Craft believe Mr. Jones will be hired?
 DETAIL QUESTIONS: QUANTITY
How many mobile phones were purchased?
How much did the consumer pay for the product?
TEST STRATEGIES
In Part 7, you must read as quickly as you can. You must also read efficiently. These strategies
will help you read more efficiently and answer the questions on Part 7 correctly.
 Read quickly and efficiently.
 Read the questions BEFORE you read the passage.
If you know what a question ask, you will have a specific purpose when you read. Look for
the answer to the question as you read.
Read these sample questions and note the information wanted.
Advertisement
How much is (a product)? Detail
What is being advertised? Main idea
Who might use the product? Inference

136
Business correspondence
When was the fax sent? Detail
What is the purpose of the letter? Main idea
What is the tone of the memo? Inference

Forms, charts, and graphs


What percentage of users are over 30? Detail
What is the purpose of the circle graph? Main idea
Who could use this information? Inference

Articles and reports


What dates are critical? Detail
What is the main idea of this article? Main idea
Who would most likely to read this report? Inference

Announcements and paragraphs


Who is (name or title)? Detail
What is the announcement about? Main idea
What is the writer’s opinion? Inference

137
PRACTICE
PART 7
Directions: In this part you will read a selection of texts, such as magazine and newspaper articles,
letters, and advertisement. Each text is followed by several questions. Select the best answer for each
question and mark the letter (A), (B), (C), or (D) on your answer sheet.

Questions 221-222 refer to the following email.

To: All Staff


From: Don Myrick

Dear Staff,

This memo serves as a reminder for changes in clock-in and clock-out times on Tuesday, December
31st and on Wednesday, January 1st. In anticipation of New Year’s Eve celebrations, all offices will
be closed from 3 p.m. on December 31st and will not reopen until 10:30 a.m. on January 1st.

Offices will be locked during this time, and only high-level personnel will have access to all
facilities.

Don Myrick
Operating President
Myrick Technologies

218. What would be an acceptable subject for this email?


(A) Offices closed until further notice
(B) Changes in job duties
(C) Change in operation hours
(D) Volunteers needed for holiday work

219. Who can enter offices after 3 p.m. on December 31st?


(A) Managers and supervisors
(B) Basic staff members
(C) Building safety guards
(D) Late-night staff members

138
Questions 223-225 refer to the following memo.

Attention All Staff

Access to the network will be unavailable on Wednesday, June 25th, from 6 a.m. to 2 p.m. for required
maintenance and testing of our network firewall system. Please plan accordingly for this outage.

If you have files stored on your network drive, please be sure to back up these files on an external drive,
as your information may be compromised during our work. We cannot guarantee retrieval of all
information after Wednesday, June 25th.

In order to best utilize our time, we ask that if you have any questions, please first go to the Frequently
Asked Questions of our website at servicedesk.com. If your question is not addressed on this list, then
contact the Service Desk at 454-334-4455 or servicedesk@mail.com.

The Service Desk Team

223. Why will the network be unavailable?


(A) A virus has been downloaded.
(B) They are replacing several old computers.
(C) They are observing national holiday.
(D) Security checks and updates are being made.

224. What are the recipients directed to do?


(A) Turn off their computers
(B) Save all files to the network
(C) Create a secondary copy of files
(D) Contact their supervisors

225. Why might recipients NOT contact the Service Desk?


(A) There is no one available to answer questions.
(B) Answers to questions have been prepared online.
(C) All questions should be directed to supervisors.
(D) The Service Desk will not answer questions on this issue.

139
Questions 226-228 refer to the following press release.

- Press release –

Downtown City Bridge to Reopen Next Month

September 10 - The town of Lincoln is pleased to announce that the Downtown City Bridge,
which has been closed for repairs for the past three weeks, is scheduled to reopen next month.

City officials were forced to close down the bridge after it failed its yearly Ministry of
Transportation inspection. The bridge's cable system had deteriorated, but has since been
replaced. The City Bridge will reopen its gates to motorists and pedestrians on the 15th of the
month.

The construction on the corner of Main Street and Division Street has also been completed.
The city was repairing a broken water main that had burst a few days ago but has since been
repaired. New traffic lights were also installed at the busy intersection during the construction
period. In addition. a part of the lane on Division Street was repaved after numerous requests
to do so.

The city expects to complete repairs on the additional three bridges in town early next year.

226. What is the main topic of the press release?


(A) The renovation of city facilities
(B) The building of a new bridge
(C) Revised inspection policies
(D) The introduction of a new cable company

227. What is NOT new at Main Street and Division Street?


(A) The road signs
(B) The traffic lights
(C) The water pipe
(D) The repaired street

228. What was the problem with the City Bridge?


(A) A new bridge has been built nearby.
(B) A part of the bridge was outdated.
(C) The width of the road was too narrow.
(D) There was not enough traffic.

140
Questions 229-231 refer to the following email.

From: Joyce Lalarna


Sent: Wednesday, September S, 9:37 a.m.
To: Thomas Sacco
Subject: Next Meeting

Hi Thomas,
I know that we had originally planned to meet on Friday to discuss the packaging your
company is designing for us, but I am afraid I have to reschedule. Phillis Louis from our
company headquarters in London will be coming that day for an unexpected visit and tour
of our Chicago Location. I then have to leave for a sales show in Las Vegas on Saturday
morning, and then I have a budget meeting in San Francisco on Tuesday. Can we look at
rescheduling for Wednesday afternoon? I will be back in Chicago then. If this won't work
for you, perhaps we could look at a time that would be more convenient. Let me know and I
will try and arrange my schedule around yours. I apologize for the late notice.

Sincerely,
Joyce

229. What is the reason for the e-mail?


(A) To propose changes to a design
(B) To ask for a marketing quote
(C) To make travel arrangements
(D) To delay a meeting

230. When will Joyce Lalarna meet with Phillis Louis?


(A) On Tuesday
(B) On Wednesday
(C) On Friday
(D) On Saturday

231. Where is Thomas Sacco's office most likely located?


(A) In Chicago
(B) In Las Vegas
(C) In London
(D) In San Francisco

141
Questions 232-234 refer to the following article.

Excellence in the Workplace

Maronnier and Felton Co. presents annual merit awards to employees who demonstrate excellence
in the workplace. These employees are nominated by Division Leaders and exemplify a high
standard of quality and innovation in their roles. This year, there are two recipients: Majumder
Singh (Business Development) and Sarah Rhee (Global Strategy).

Mr. Singh has been with Maronnier and Felton for 5 years, and brings over 15 years of experience
as an independent financial manager. During his time with the firm, he has helped increase profit
margins by almost 10 percent. In particular, he has successfully developed new financial programs,
establishing new accounts and managing the portfolios of some of the firm's highest profile clients.
Ms. Rhee's achievements are of equal note. Even though she joined the firm only 2 years ago, she
has made significant contributions in the creation and successful launch of our Global Strategy
Operations Team. This team is responsible for the firm's expansion into the Australasian region.
She negotiated several key partnerships that ensured a smooth entry into key markets. Furthermore,
she played a key role in introducing several top-performing managers and strategists to the
Maronnier and Felton team. In the past fiscal year, she also launched a new global collaboration
intraner that has helped improve manager performance by an estimated 20 percent.
Prior to joining the firm, she spent 15 years managing a successful retail sales company.
We thank Mr. Singh and Ms. Rhee for their exceptional contributions.

232. What is the purpose of the article?


(A) To introduce new employees
(B) To inform employees of the new management intranet
(C) To announce award recipients
(D) To request sales figures for the last quarter

233. What does the article say about Mr. Singh?


(A) He worked as a fashion consultant.
(B) He nominated Ms. Rhee for an award.
(C) He is an important customer of the company.
(D) He has contributed to the success of the company.

234. What is NOT true about Ms. Rhee?


(A) She has brought a lot of important new managers.
(B) She helped to develop the global management intranet.
(C) She has been at the company longer than Mr. Singh.
(D) She has a lot of experience in business development.

142
Questions 235-238 refer to the following form.

Just Add Water


A Florist with YOU in Mind!

Ordered by: Ship to: (if different from ‘Ordered by’)


Name ____________________________ Name ____________________________
Address ____________________________ Address ____________________________
City ________________________ State ______ City ________________________ State ______
Email ____________________________ Gift message ____________________________
Phone ____________________________

If paying by credit card, please make sure your name and address match the information on your
credit card statement.

Order Form:
Page # Quantity Item # Description Total

Subtotal
Method of Payment: Shipping & Delivery (+$8 each)
Check ____ Money Order _____Credit card (see below) Overnight (+$25 each)
Credit Card # _____________ Total
Expiration Date _____/ _____
Authorized Signature ____________

Unconditional Guarantee:
Your complete satisfaction is our main priority. If you are unsatisfied in any way, contact us within
24 hours of receipt and we will promptly replace or refund your order.

How to Order:
Online: www.justaddwater.com
Ordering online is faster and easier than ever on our safe and secure website. Just click on the
picture of arrangement you desire, fill in your billing and shipping information, and click
“Purchase” you can even save your information to make ordering faster next time!

Phone: 1-888-444-222 Fax: 1-888-444-2221


Email: orders@justaddwater.com
Mail: Just Add Water
424 W. Main Street
Gary, IN 33244

143
235. What can be inferred about ordering online?
(A) This is option is not available at this time.
(B) They must be made 24 hours in advance.
(C) Only certain arrangements are available online.
(D) The process has been improved.

236. What is not a requirement of credit card purchases?


(A) A valid form of identification
(B) Matching physical address
(C) Information on the card’s front
(D) The cardholder’s signature

237. Beyond the cost of the flowers, what other charges are there?
(A) The vase and bow
(B) Shipping and transportation
(C) Message delivery
(D) Government fees

238. Why would the form need two addresses?


(A) The purchaser and recipient are the same.
(B) The purchaser and the recipient may differ
(C) The purchaser is paying via check.
(D) It is an online order.

144
Questions 239-242 refer to the following email.

Dear Employees

Earlier today, we publicly reported our year-end financial results. Specifically, we reported a
combined total operating revenue for last year of $442 million, compared to $393 million in 2011.
Additionally, we reported the operating income of $12 million and non-operating income of $7.9
million, for a combined income total of approximately $20 million.

These solid results serve as a testament to our disciplined approach in helping to control the rate of
increase of our overall expense trend. Last year, we held our rate of expense growth to 4.8%, which
is below hospitality industry standards. This is due to the dedication all of you have demonstrated
over the last year.

It appears there has been a consistent revenue growth trend since last August, despite the slowly
recovering economy. It is very encouraging to return to growth, especially after the last few years of
little growth. Revenue growth fuels our ability to make the necessary investments in our facilities
and people and validates our value proposition of providing a high-quality lodging experience that
is affordable.

It is abundantly clear, from the items you see below, that we are beginning to reap the dividends
from the investments we have made in our business over the last few years.

 This January, 6 Starlight Hotels were named Leapfrog Top Hotels, an honor that measures
outstanding success in areas such as room service, business center service and restaurant
outlets and safety practices.
 We led the nation among all registered three-star hotel chains with top rankings in 15 quality
measure categories by National Travel and Hospitality Associations.
 Last year we opened one corporate-owned hotel and added three new franchisee hotels to
our chain.
 We now have a record of 1,100 employees working across 18 hotels and resorts in unit-
based teams, actively engaged in improving quality and service across the organization.

This year, we are certain to be beset with opportunities and challenges, both new and old. Our own
goal to grow to a 30-hotel chain in five years shows that now is not the time to let up on our efforts.
We are on the right path, executing the right strategies and making appropriate adjustments as we
continue to better understand our opportunities and market.

We really appreciate your staying on course and focused this year and beyond.

Thank you for hard work.

Burt G. Mandelson
Chief Finance Officer

145
239. Which is one of the purposes of the email?
(A) To offer constructive critique on an employee's performance
(B) To announce financial incentives for long-term employees
(C) To inform employees of an ongoing criminal investigation
(D) To list achievements accomplished in the previous year

240. The word “solid” in paragraph 2, line 1 is closest in meaning to


(A) substantial
(B) compact
(C) pure
(D) vivid

241. Which performance measure was better than the industry average according to the
email?
(A) Foreign investment
(B) Cost-cutting
(C) Revenue increase
(D) Tax savings

242. Which statement is correct about Starlight Hotel Group?


(A) A total of 30 Starlight Hotels are currently open worldwide.
(B) Some Starlight Hotels received a high degree of recognition last year.
(C) It recently acquired an exotic seafood restaurant chain called Leapfrog.
(D) It recently opened several facilities run by corporate management.

146
Questions 243 – 245 refer to the following letter.

243. Who owes money?


(A) No one
(B) Ms. Tomkins
(C) China Books
(D) The author

244. When was the check written?


(A) April 23
(B) May 30
(C) October 13
(D) October 23

245. According to the letter, which of the following is NOT true?


(A) Ms. Tomkins has paid twice.
(B) This is Ms. Tomkins' second letter.
(C) Ms. Tomkins has received two notices.
(D) The company received the payment

147
Questions 246-247 refer to the following pay slip.

ABC Staffing
4458 Company Dr.
Newcombe, OH 88838

Pay Period: from 9/1/2016 to 9/30/2016


Pay Date: 9/30/2016

Employee: Morgan Carr Morgan Carr


Department: 0034 134 Oak St.
Marital Status: Married Newcombe, OH 88838
Number of Allowances: 01

Hours and Earnings Taxes and Deductions


Description Hours Rate This This year Description This This year
Period Peiod
Regular 20.00 $21.52 $430.4 $1,068.40 State Tax
$24.32 $60.88
Other $552.12
Total Deductions Not Pay This
Gross Pay This Year Gross Pay This Period
This Period Period
$1,620.52 $430.40 $24.32 $406.08

246. How often does Morgan Carr get paid?


(A) Every month
(B) Every week
(C) Every other week
(D) At random

247. What kind of information is NOT found in the document?


(A) Employee’s name
(B) Employer’s address
(C) Supervisor’s name
(D) Employee’s hourly rate

148
Questions 248-250 refer to the following article.

What Comprises Your Credit Score?

When applying for a student loan, buying your first car, or buying a home, there is one major
factor that can hinder you, and that is your credit score. Below are the 5 major factors that
affect your credit score. Keep these items in check, and you’ll be driving off the lot in your
new set of wheels without a single hitch!

1. Payment History: This includes paying your bills on time, any debts that have been sent to
collections, and any official action like bankruptcy or court judgments that have been made.

2. Money Owed: Any debt that is currently a part of your porfolio affects your credit score.
Most banks will only allow for debt to equal up to 30% of your yearly income.

3. Established Credit: While you can’t have too much debt on record, it’s also important that
you demonstrate, you can be responsible with debt. If you have never had a credit card before,
it will be hard to be approved for a large loan.

4. Recent Activity: If you have opened a line of credit within the past 30 days, banks may
question your ability to pay all your debt.

5. Types of Debt: It is important to demonstrate that you can handle not only debt but also
different types of debt, like credit cards, mortgages, and personal loans.

248. What is the topic of the article?


(A) Buying a car
(B) Elements of credit
(C) Good and bad credit
(D) Loan applications

249. What kind of information is included in payment history?


(A) Evidence of good judgments
(B) How many credit cards you have
(C) People who own your money
(D) Loans you didn’t repay

250. Where can one demonstrate financial consistency?


(A) Payment history
(B) Money owed
(C) Established credit
(D) Recent activity

149
Questions 251-253 refer to the following email.

Dear Students,

It is quite important to read directions before you begin an assignment – actually, before you begin
anything! I can’t tell you how many times I have to give an assignment a score of 0 simply because
the student didn’t follow directions. This has been an issue especially with your class, so I am going
to give you an assignment to see how well you follow directions.

Compose an email to me. In the subject line, write your full name and the title of this class. In the
body of the email, write a well-organized paragraph telling me about a time you followed directions
and were rewarded for it or didn’t follow directions and has consequences to pay. At the end of the
paragraph, give yourself a grade on a scale of 1-10. Your parargraph, depending on the length and
thoughtfulness, will earn between 30 and 50 extra credit points!

As is the theme of this announcement, all directions must be followed in order to earn your extra
credit points. Good luck and enjoy!

Professor Gary

251. Which of the following is an instruction given?


(A) Complete an application for a new job
(B) Skip over wordy passages
(C) Spend time reading all details
(D) Email a friend about following directions

252. What is a consequence of NOT following directions according to the passage?


(A) Retaking the exam
(B) Getting a lower grade
(C) Wasting a lot of time
(D) Earning extra credit

253. What is the point of the email?


(A) Professor Gary has noticed a disturbing trend.
(B) Professor Gary will not award extra credit points.
(C) This class is difficult, and students must work hard.
(D) The final exam has been rescheduled.

150
Questions 254-256 refer to the following recipe.

Chicken Pinwheel
Serves: 6
Time: 5 minutes preparation, 45 minutes cooking

Ingredients

 12 thin chicken breast fillets (or 6 chicken breasts, butterflied and cut in two)
 24 slices dried beef
 12 ounces sour cream*
1 can cream of chicken soup (or cream of mushroom soup)*
 6 servings prepared rice**
 1 pound asparagus

Directions
1. Preheat oven to 425 degrees Fahrenheit.
2. Lay out chicken breast fillets. Layer 2 slices of dried beef on each fillet so all of the chicken is
covered. Roll fillet from top to bottom and place in an 8-inch x 8-inch glass baking dish.
3. Stir together sour cream. Soup, and salt and pepper in a medium bowl. Pour the mixture over
chicken pinwheels.
4. Bake chicken dish at 425 degrees Fahrenheit for 20 minutes. Then decrease baking temperature
to 375 degrees Fahrenheit and bake for another 20-25 minutes. Cut into the middle of a chicken
pinwheel to ensure the meat is thoroughly cooked before removing from oven.
5. Meanwhile, prepare rice and asparagus.
6. To serve, lay a bed of rice on the plate, top with 2 pinwheels, spoon sauce over, and balance 4
asparagus spares on top. Bon appetite!

* Low-fat ingredients can be substituted.


** Other starches, like potatoes or pasta, can be used.

254. Which of the following is NOT an option suggested by the recipe?


(A) Serve over a bed of spaghetti
(B) Substitute bacon for the dried beef
(C) Use 50% reduced-fat soup
(D) Use vegetarian soup
255. What ingredient is missing from the list?
(A) Seasonings or spices
(B) Green vegetables
(C) Dairy products
(D) Meat
256. How is the meal cooked?
(A) Deep-fried
(B) Grilled
(C) Baked
(D) Boiled

151
Questions 257 – 261 refer to the following article and email.

Pendale Mobile Networks Unveils New Cell Phone


By Ray Sanders

Seoul, March 12
The most recent cell phone to be produced by Seoul-based Pendale Mobile is certain to make a big
splash in the mobile phone industry. Featuring an unusually large display screen, online capabilities,
and massive memory storage space, Pendale’s H-989 offers the user a phone, computer, and
personal assistant, all wrapped up in a portable package.
The new product’s easy-to-use touch screen makes it appealing to users tired of having to carry
around operations manuals with them. It will also come with high speed Internet, offering e-mail
and web-surfing features, in addition to being able to quickly send and receive information.
In a press conference last week, H-989 developer Sung Min Park stated that he was overwhelmed
with all of the attention the new phone has received. He said that while demand may exceed supply
for a short period of time after the product is released to the public, company executives will work
tirelessly to ensure that the H-989 is available to all who wish to purchase it.

The H-989 will be available to the general public in the beginning of May
To: Mathew Oppenheim
From: Niko Gomez
Date: March 20
Subject: New Cell Phones

Mathew,

As part of our yearly supplies updates here at Amerald Co., I have told the manager of our sales
department to order 45 cell phones for our sales representatives by April 1. I’d like to suggest that
we order the new Pendale Mobile H-989 as the product seems to meet our needs. The Popular
Electronics website has a feature on the H-989 that you should take a look at.
As you are aware, our sales representatives spend a great deal of time on their cell phones as they
travel between clients. As it stands now, they are equipped with more traditional phones. However,
if we were to offer each of them an H-989, I expect that sales would increase as a result of the
phone’s capability to function more quickly and efficiently than older types.

Please get back to me as soon as possible regarding the H-989. I will be traveling to China for the
next three days, but I’d like to get together with you to discuss this when I return. Upon your
approval of this purchase, I’d like to submit an order to Pendale
Mobile as soon as the model becomes available.

Regards,
Niko

152
257. What is NOT mentioned as a feature of the H-989?
(A) It has a large display screen.
(B) It can access the Internet.
(C) It has a long battery life.
(D) It is easy to use.

258. What is inferred about the H-989?


(A) It is the most expensive phone on the market.
(B) It will be discounted for a certain period.
(C) It is not yet available in stores.
(D) It will come in 6 different models.

259. What is the purpose of the e-mail?


(A) To suggest the purchase a product
(B) To request permission to attend a meeting
(C) To ask for available funds to order supplies
(D) To compare newspaper articles

260. What does Niko Gomez imply in the e-mail?


(A) The price Pendale Mobile is asking is too high.
(B) Amerald Co. will release a new cellular phone model.
(C) Mathew Oppenheim will visit China.
(D) Amerald Co.'s sales representatives need new phones.

261. When does Niko Gomez hope to contact the Pendale Mobile Company?
(A) In February
(B) In March
(C) In April
(D) In May

153
Questions 262-264 refer to the following email.

To: chartes@danone.com
From: jgibson@katpine.net
Subject: Upcoming visit
Dear Charles,
The purpose of this letter is to confirm the travel dates for Sharon and myself. We had some
difficulty working out our schedules but have finally agreed that the best time for our trip would be
from February 14 to 21. We will be traveling to France by Air France, and then we will proceed to
Switzerland by train three days later, whereupon we will leave for home after we've had plenty of
chances to catch up on old times.
Please note below the times and dates of our schedule:

Travel itinerary:
February 14 - Air France departure point: New York, New York
February 15 - Air France arrival point: Paris, France
February 17 - Eurostar departure point: Paris, France
February 18 - Eurostar arrival point: Zurich, Switzerland
February 20 - Swiss Air departure point: Zurich, Switzerland
February 21 - Southwest Airlines arrival point: New York, New York
As you can see from our schedule, we will be changing destinations twice. During our travel, we
were hoping to rent a car in each city. If you could arrange that one be sent to the airport with a
driver on our arrival in each city, that would be greatly appreciated. I look forward to getting
reacquainted with you after such a long time.
Best regards,
John
262. What does this e-mail confirm?
(A) A car has been rented for John.
(B) Accommodation arrangements have been taken care of.
(C) The dates of travel have been set.
(D) The plane tickets have been reserved.

263. What is being requested by the writer?


(A) To be picked up at the airport
(B) To have transportation arranged
(C) To send the plane tickets right away
(D) To convince Sharon to go on the trip
264. What does the letter imply about the relationship between Charles and John?
(A) They will be meeting in France for the first time.
(B) Charles is John’s superior.
(C) They have not seen each other for a long time.
(D) John can’t wait for Charles to meet Sharon.

154
Questions 265-269 refer to the following letter and receipt.

155
265. What is the letter mainly about?
(A) Requesting exchange for receiving different products
(B) Ordering products from a wig company
(C) Asking for money back for products ordered
(D) Complaining about the quality of the products

266. What can you tell about Mr. Pulse?


(A) He is excited to receive his products.
(B) He is anxious to sell the products.
(C) He is very angry for the bad quality of hair.
(D) He is upset for not receiving the products.

267. What is the result of Wig Hair company failing to deliver the products?
(A) He lost a few customers.
(B) He wasted lots of money for nothing.
(C) His company cannot sell any hair products anymore.
(D) Most of his customers will not buy hair products from his store anymore.

156
268. When was the order placed?
(A) October 3rd
(B) October 12th
(C) September 1st
(D) September 13th

269. What is most likely to happen in the future?


(A) He will try to visit Wig Hair to see their new products.
(B) He will introduce other stores to Wig Hair.
(C) He will not buy any of Wig Hair products.
(D) He will sue Wig Hair for their terrible service.

Questions 270-272 refer to the following passage.

Opportunity for International Experience


Currently we have four positions open for middle managers who wish to participate in it six-month,
international exchange program. The program will involve sharing with our counterparts in the
respective country best practice principles learned while on the job here in our head office. During
your brief excursion, you will spend a considerably amount of time learning about the domestic
branches’ markets, with a specific focus on market research in the area of domestic consumers'
preferences. Afterwards, a comprehensive report of our research results will: be submitted to the
head office on your return.
Interested applicants must fill out an application in addition to submitting a maximum two-page
essay on the benefits of participating in an international exchange and how that experience can add
value to the corporation. The deadline to apply is October 31

270. What is the purpose of the notice?


(A) To announce the promotion of some managers
(B) To invite staff members to the international conference
(C) To request a report on the domestic market
(D) To recruit people interested in participating in a special program

271. What must be turned in at the end of the six-month period?


(A) An essay
(B) An application form
(C) A report
(D) The course fee

272. Which of the following is NOT required to be done by the applicants?


(A) Complete a form
(B) Write about what they have learned
(C) Provide a reason for their application
(D) Get permission from their immediate managers

157
Questions 273 – 277 refer to the following passages

Overseas American Bank Card type: Gold Visa PO Box 551-4433 Account holder:
John White
New Jersey, Account number: 5444 1234 5998 3888
NY 28891-4433
April statement for account activity from March 1,2006 through March 31, 2006
(Page 1 of 2)

Basic information
Previous balance: $299.42 Credit line: $5,800
Last payment made: March 10, 2006 New charges: $805.25
Last payment amount: $299.42 Available credit: $4,994.75
Finance charges: $0.00 Minimum Payment Due: $35.00
Payment Due Date: April 20, 2006
Detailed Transaction Summary
This Month’s Purchases
Merchant Transaction Date Amount
Times Book Store (New York, NY) March 3,2006 $35.80
The Marche (Santa Barbara, CA) March 8,2006 $59.90
The French Restaurant (Los Angeles, CA) March 14, 2006 $54.15
Good Sound Music (Eureka, CA) March 16,2006 $476.00
Subtotal$625.85
New Cash Advances
Location Transaction Date Amount Cash Advance Fee Total Charge
Country NICE ATM March 25,2006 $77.00 $2.40 $79 40 (San Diego, CA)
Peak Tower ATM March 15,2006 $97.00 $3.00 $100.00 (Eureka, CA)
Subtotal $179.40
TOTAL NEW CHARGES $805.25
Please Note
» Kindly refer to Page 2 for detailed finance charges.
» You may visit our website at: www.oab.com or call us at 1 -800-988-8888 for:
•Request for account history
• Change of billing address
• Other account related matters

158
273. What sort of document is this?
(A) A credit card statement
(B) A bank transaction record
(C) An accounting statement
(D) A withdrawal form

274. What is the smallest payment the account holder must pay by April 20, 2006?
(A) $35.00
(B) $179.40
(C) $299.42
(D) $805.25

275. What is known about the account holder?


(A) He has been spending his money unwisely.
(B) He traveled to many cities in March.
(C) He likes to read while traveling.
(D) He is very fond of exotic foods.

276. What should Mr. White do if he wishes to have his bill sent to a new location?
(A) Write a letter to the New Jersey office
(B) Call the toll-free number
(C) E-mail the account manager
(D) Visit the nearest American Bank

277. On which date did Mr. White withdraw the most amount of money from an ATM?
(A) March 3, 2006
(B) March 16, 2006
(C) March 25, 2006
March 15, 2006

159
Questions 278 - 281 refer to the following passage.
Business travelers find that some jobs take them away from home for longer than a few
days at a time. Those who find themselves at a new job site for weeks or even months
often find it more comfortable and economical to stay at an apartment-hotel rather than
a traditional style hotel. The comfort and convenience of these short-term residences
are making them more and more popular among frequent business travelers. They
provide advantages that more luxurious traditional hotels do not. Apartment-hotels
offer both small and full-size apartments that are available to rent on a weekly or
monthly basis. Apartments are fully furnished with everything from sofas and writing
desks to dishes and silverware. They also usually include cable TV service and Internet
access. Best of all, they are run like hotels, with cleaning and linen services, exercise
rooms and restaurants, and a desk clerk to take messages and help tenants with
questions about the city. In addition, the prices are much more reasonable than the rates
normally charged at a traditional hotel. One of the reasons that many apartment-hotels
are economically priced is that they are often not found in a city's downtown area. This
is not necessarily a disadvantage, however. They are usually conveniently located near
public transportation, so the expense of a rental car is not always necessary. In short,
apartment-hotels offer a convenient alternative to the business traveler, as they are
more cost- effective than traditional hotels and more comfortable than hastily furnished
apartments. Many business travelers find that apartment-hotels are as comfortable as it
is possible to be away from home.

278. Why are these residences called "apartment-hotels"?


(A) They have characteristics of apartments and hotels.
(B) They contain full-sized apartments.
(C) They look like hotels.
(D) They have only short-term tenants.

279. Who would be likely to use an apartment-hotel?


(A) A businessperson on an overnight trip
(B) A family of tourists
(C) An engineer on a ten-week project away from home
(D) A consultant in town for a convention

280. What is NOT mentioned as an advantage of apartment-hotels?


(A) They are furnished.
(B) They have cleaning service.
(C) They are centrally located.
(D) They have a desk clerk on duty.

281. How do apartment-hotels compare with standard hotels?


(A) The rooms are larger.
(B) They are not as comfortable.
(C) There are fewer services offered.
(D) They are less expensive for a long stay

160
Questions 282 – 283 refer to the following letter.

From:atompson@wirefee.c
om
To:sgoodwin@gmail.net
Subject: Workshop date
Date: Sept 12

Dear Sandra,

Thanks for your e-mail. Preparations for the workshop are challenging, and our staff has
been working diligently to make it successful. We hope to emulate the success of our
last workshop. Unfortunately, I’m afraid the workshop must be rescheduled, as our
featured speaker cannot make the October 14 date. Please note that the workshop date
has been changed to November 6 to accommodate our speaker. We are doing our best
to inform all participants as soon as possible of this change. Our website has been
updated accordingly, and I hope this doesn’t cause any conflicts in your schedule.

Thanks,

Amy Thompson

282. Why was the e-mail sent?


(A) To cancel travel arrangements
(B) To request workshop registration forms
(C) To congratulate a colleague
(D) To change the date of an event

283. When will the event be held?


(A) On August 12
(B) On November 6
(C) On September 12
On October 14
Questions 284-288 refer to the following passages.

161
162
284. Why does Lim apologize in his e-mail?
(A) A black-out period changed his travel plans.
(B) He was busy on the weekend of the 3rd.
(C) He forgot to book his return flight.
(D) It is his first time traveling to Tokyo.
285. How does Lim plan to get to the hotel from the airport?
(A) He will take a shuttle bus.
(B) He will hire a taxi.
(C) He will walk the short distance.
(D) He will be picked up.
286. What type of travel was booked?
(A) A business trip
(B) A vacation
(C) A group tour
(D) A study trip
287. How does Lim's reservation differ from the original ad?
(A) He decided to stay in a 4-star hotel.
(B) He requested a room with a television.
(C) He switched to a double room.
(D) He chose a different travel agency
288. How was this trip paid for?
(A) With travel points
(B) With a credit card
(C) With cash
(D) With a check

163
Questions 289-293 refer to the following letter and table.

Welcome to the Washington Heights Memorial Hospital


Residency Program!

We are excited to join you as you embark on your long sought after medical career. We have a
program here that we are very proud of for the way it prepare you to become efficient and creative,
contributive members of the medical community.
In your enclosed welcome pocket, you will find a rotation schedule that details which units you will
work in each year of your residency, a detailed list of medical staff and their specialties here at
Washington Heights, all important contact persons and their contact information, as well as a
welcome packet to the city of Washington Heights including local restaurants, attractions, historical
sites, and housing options.
We look forward to all the learning and experiences that will be gained as you spend the next four
years with us.

Sincerely,
Dr. John Roberts, M.D., P.C.

1st-Year Residents
General Surgery, Vascular Surgery, Pediatrics, Plastic Surgery, Thoracic 5 months
Surgery, Surgical Oncology
Otolaryngology/Head & Neck Surgery 3 months
2nd-Year Residents
Intensive Care Ward 6 months
Children's Ward 2 months
Washington Heights Emergency Center 4 months
3rd-Year Residents
Research 4 months
Maternity Ward 4 months
Washington Heights Emergency Center 4 months
4th-Year Residents
**Resident's Choice** 4 months
Washington Heights Emergency Center 4 months
Otology/Facial Plastic Surgery 4 months

164
289. What happens in the first 4 months of residents’ last year?
(A) They will have a rotation in emergency medicine.
(B) They spend time in the choice department.
(C) They can focus on an area of interest.
(D) They focus on research.

290. What is NOT in the resident welcome packet?


(A) Lists of local venues
(B) A staff directory
(C) An apartment guide
(D) Grading procedures

291. What role does Dr. Roberts probably play?


(A) Head of the residency program
(B) Chief heart surgeon
(C) Director of hospital marketing
(D) Head resident

292. Which document is the table, as listed in the welcome letter?


(A) A list of medical staff
(B) Contact persons
(C) A list of local restaurants
(D) The rotation schedule

293. Which department is most valued for training residents?


(A) Children's Ward
(B) Intensive Care Ward
(C) Emergency Center
(D) Research Center

165
Questions 294-298 refer to the following article and recipe.

Why It's Good to Be Vegetarian


Vegetarians are those among us who have decided to remove meat from their diets. While scientific
studies concerning the benefits of living a vegetarian lifestyle are scarce, there are a few benefits that
those of the general medical population agree upon:

It lowers your Body Mass Index (BMI).


Because a vegetarian diet is inherently lower in fat due to the fact that all animal protein fats are
absent, vegetarians usually have a BMI 25% less than those of us who indulge in meat dishes. While
vegetarians most often choose their lifestyle for reasons concerning animal cruelty, reasons
concerning lowering one's BMI is the next most prevalent reason for choosing a vegetarian lifestyle.

It lowers your Cholesterol.


Medical research increases its focus on heart attack and other cholesterol-related illnesses and ill-
effects, and so does the percentage of vegetarians among the population. Studies show that average
individuals can decrease their cholesterol level by.5 mmol/l by converting to vegetarianism.

It increases your Lifespan.


While sound scientific reasoning has yet to explain this phenomenon, people who live a vegetarian
lifestyle beginning no later than their 30s live longer than those who do not. Women live an average
of 7 years longer and men live an average of 5 years longer.

Stuffed Peppers
Ingredients:
• 4 teaspoons olive oil
• 2 poblano-chiles, seeded and chopped
• 1 jalapeno, chopped
• 1 medium onion, chopped
• 1 clove garlic, crushed with garlic press
• 4 large bell peppers
• 1 lb. chicken breasts (or substitute with tofu)
• 1 teaspoon cumin
• Salt
• 2 cups fresh corn kernels
• 1/2 cup shredded Monterey jack cheese
• 1/4 cup loosely packed fresh cilantro leaves, chopped

Instructions:
1. In 12-inch skillet, heat 2 teaspoons oil on medium heat until hot. Add poblanos, jalapeno,
and onion, and cook for 8-9 minutes or until vegetables are tender, stirring occasionally. Add
garlic and cook 1 more minute. Transfer to a bowl.

166
2. Meanwhile, cut off the top of each pepper and scoop out seeds and membranes. Transfer
peppers to microwave-safe plate and cook on high heat for 3 or until soft.
3. In the same skillet, add remaining 2 teaspoons oil, cubed chicken or tofu), cumin, and ½
teaspoon salt and cook on medium-high heat about 10 minutes or until tofu has a golden
crust. Stir in mixture from bowl, corn, cheese, and cilantro; heat through.
4. To serve, spoon mixture into pepper shells. Arrange on a platter and enjoy.

Recipe Nutrition Information:


With Chicken With Tofu
Fats 15 grams 7 grams
Sugars 3 grams 5 grams
Sodium 12 grams 12 grams
Protein 10 grams 10 grams

294. What is the challenge with vegetarianism mentioned in the article?


(A) Each vegetarian is very different from another.
(B) Systematic research about it is rare.
(C) Vegetarians are socially discriminated.
(D) Being vegetarian is very costly.

295. What is a benefit of ridding meat from your diet?


(A) It increases muscle and ligament health.
(B) It decreases blood-related disorders.
(C) It decreases your body mass index.
(D) It increases your activity level.

296. What provision makes the stuffed peppers recipe vegetarian?


(A) The abundance of vegetables
(B) The replacement of protein source
(C) The absence of eggs
(D) The addition of cumin
297. What idea in the article is supported by the recipe's nutrition information?
(A) Salt content is the same in meats and tofu.
(B) Tofu is unhealthy due to increased sugar content.
(C) Tofu and meat contain equal amounts of protein.
(D) Tofu provides a healthier substitute for meats.

298. What is NOT a benefit of a vegetarian lifestyle?


(A) Vegetarians weigh less.
(B) Vegetarians are healthier.
(C) Vegetarians don't need cholesterol medications.
(D) Vegetarians live more active lifestyles.

167
Questions 299-300 refer to the following advertisement.

Blade O Matic Food Processor


$99.99

What’s Included
- Blades included: all-purposed blade, dough hook, slicing disk, shredding insert
- 5-foot electrical cord
- 2 storage containers

Product Description
Slice, shred, and chop your way to culinary perfection – or just simplify the steps in preparing your
family’s favorite vegetable stew. The Blade O Matic Food Processor belongs in every home: slicing
fruits and vegetables, kneading bread dough, and chopping all things edible in its 8-cup work bowl
that is rated to be used with extremely hot foods and can also be stored in cold environments. It is
also dishwasher safe and shatterproof.

299. What is NOT a part of the Blade O Matic?


(A) A utensil for preparing baked goods
(B) A power supply cable
(C) A portable cooler to keep goods cool
(D) A couple of receptacles

300. What is true of the work bowl?


(A) It has a 6-cup capacity.
(B) It is freezer safe.
(C) It melts easily.
(D) It can be hand-washed only.

-END-

168

You might also like